Thread Rating:
  • 0 Vote(s) - 0 Average
  • 1
  • 2
  • 3
  • 4
  • 5
need nbme form 3 and 6 with answers please - sameerajs
#1
can somebody e mail me at sameerajs thanks! alot
Reply
#2
can you send me them too at heroangel007@hotmail,com
thanks and best luck
Reply
#3
http://www.doctorshangout.com/group/usml...t%3A173725&groupId=2002836%3AGroup%3A2682
Reply
#4
Block 1




1.) A 50-year-old man with a 20-year history of type
2 diabetes mellitus has had sensory neuropathy for 2 weeks.
Pulses are decreased at the ankle. There is no peripheral edema. His serum
glucose levels have been between 150 mg/dL and 200 mg/dL over the past 6
months. Which of the following is the most effective measure to prevent
serious foot infections?

A) Use of support hose

B) Use of well-fitted shoes

C) Prophylactic antibiotics

D) Daily aspirin and dipyridamole

E) Decrease in serum glucose level


2.) A 63-year-old woman comes to the emergency
department because of palpitations for 1 hour. She was diagnosed with
hyperthyroidism 2 months ago but has not received treatment. Three
hours before arrival, she drank two bottles of beer with dinner. Her blood
pressure is 90/60 mm Hg, and pulse is 85/min and irregularly irregular.
Examination shows a variation in intensity of S1. There is an absence of
waves in the venous pulse. An ECG is shown. Which of the following is
the most likely cardiac rhythm disturbance?

A) Atrial fibrillation

B) Atrial flutter

C) Multifocal atrial tachycardia

D) Paroxysmal atrial tachycardia

E) Second-degree atrioventricular block, Wenckebach type

F) Sick sinus syndrome

G) Sinoatrial exit block, type II

H) Torsades de pointes

I) Wandering pacemaker

J) Wolff-Parkinson-White syndrome


3.) A 5-year-old girl is brought to the physician 30 minutes after
being bitten on the forearm by her cat. Examination shows a small
puncture wound covered with dried blood. She is at increased risk of
infection for which of the following reasons?

A) Arm wounds are more susceptible to infection

B) Salivary enzymes in cats augment infection

C) It is difficult to cleanse pathogens from this wound

D) The Staphylococcus species involved is particularly virulent

E) Young children are less able to resist infection than adults


4.) A 4-year-old girl is brought to the physician because of pallor
and jaundice for 2 days. She had previously been well, although she was
treated for jaundice with phototherapy for 2 weeks while a newborn.
Her mother and two additional maternal relatives underwent splenectomy
during childhood for unknown reasons. Examination of the patient shows
jaundice. The spleen tip is palpated 4 cm below the left costal margin.
Which of the following blood smear findings is most likely to explain this
family's condition ?

A) Elliptocytes

B) Howell-Jolly bodies

C) Schistocytes

D) Sickle cells

E) Spherocytes


5.) A sexually active 20-year-old woman has had fever, chills, malaise, and pain of the
vulva for 2 days. Examination shows a vulvar pustule that has ulcerated and
formed multiple satellite lesions. Nodes are palpated in the inguinal and
femoral areas. A smear of fluid from the lesions establishes the diagnosis.
Which of the following is the most likely causal organism?

A) Chlamydia trachomatis

B) Haemophilus ducreyi

C) Neisseria gonorrhoeae

D) Streptococcus pyogenes (group A)

E) Treponema pallidum


6.) A 5-year-old boy is brought to the physician 1 hour after
urinating bright red blood. He has been taking ibuprofen since injuring his
right flank while wrestling with friends yesterday; he also has been
taking penicillin for 3 days for streptococcal pharyngitis. His
temperature is 36.7 C (98 F), blood pressure is 90/48mm Hg, pulse is 108/min,
and respirations are 18/min. Examination shows purple ecchymoses over
the shins and right flank; there is tenderness of the right
costovertebral area. The abdomen is nontender. Genital examination shows no
abnormalities. There is no edema. Urinalysis shows gross blood;
microscopic examination shows 5–10 leukocytes/hpf and erythrocytes that
are too numerous to count. Which of the following is the most likely
explanation for this patient's hematuria?

A) Acute pyelonephritis

B) Ibuprofen-induced acute papillary necrosis

C) Post-streptococcal glomerulonephritis

D) Rhabdomyolysis

E) Traumatic injury to the kidney


7.) A 16-year-old girl comes to the physician for her first prenatal
visit at 12 weeks' gestation. She has not had any immunizations since
the age of 5 years. She has received the following immunizations at the
recommended ages:

5 Diphtheria-tetanus-pertussis
3 Hepatitis B
1 Measles-mumps-rubella
4 Oral poliovirus

Examination shows no abnormalities. Which of the following is the most
appropriate immunization to administer at this visit?

A) Diphtheria-tetanus-pertussis

B) Diphtheria (child)-tetanus toxoid

C) Diphtheria (adult)-tetanus toxoid

D) Haemophilus influenzae type b

E) Hepatitis B

F) Inactivated poliovirus


8.) A 12-year-old boy is brought to the physician because of
temperatures to 40.2 C (104.4 F), lethargy, and a stiff neck for 2 days. A
diagnosis of pneumococcal meningitis is made, and the child is admitted to
the hospital for antibiotic therapy. Serum studies 24 hours after
admission show:

Na+ 117 mEq/L
Cl– 89 mEq/L
HCO3– 25 mEq/L

Which of the following is the most likely explanation for these findings?

A) Excessive cortisol release

B) Hyperaldosteronism

C) Inappropriate secretion of ADH (vasopressin)

D) Increased production of leukotriene C4


9. )An asymptomatic 57-year-old man with a 3-year history of type 2
diabetes mellitus comes for a routine follow-up visit. Examination shows no
abnormalities. Serum studies show:
Aspartate aminotransferase (AST, GOT) 76 U/L
Alanine aminotransferase (ALT, GPT) 86 U/L
Iron 260 µg/dL Total iron-binding capacity 300 µg/dL (N=250–450)
Ferritin 1200 ng/mL Antinuclear antibody negative
Serologic testing for hepatitis is negative. Which of the following is
the most appropriate next step in management?

A) Corticosteroid therapy

B) Interferon therapy

C) Penicillamine therapy

D) Chronic phlebotomy

E) No therapy indicated


10.) A 26-year-old man has had anxiety and insomnia since he lost his
job 1 month ago. He also has had palpitations, daily headaches, and
flatulence. He has no history of psychiatric disorders. His mother has
a history of bipolar disorder, mixed, and his father has alcoholism;
his younger sister had a history of drug abuse but has been abstinent
from drugs for 3 years. Which of the following is the most likely
diagnosis?

A) Adjustment disorder

B) Bipolar disorder, depressed

C) Generalized anxiety disorder

D) Major depressive disorder

E) Post-traumatic stress disorder



11.) A previously healthy 32-year-old man comes to the emergency
department because of a 12-hour history of fatigue and shortness of breath
at rest. He takes no medications. His temperature is 7 C (98.6 F), blood pressure is
96/58 mm Hg, pulse is 110/min, and respirations are 22/min. Examination shows
scleral icterus and conjunctival pallor. There is no hepatosplenomegaly. Laboratory
Studies show:
Hemoglobin 6.2 g/dL
Serum Leukocyte count 8800/mm3
Bilirubin, total 8.5 mg/dL
Reticulocyte count 16%
Direct 1.5 mg/dL
Platelet count 245,000/mm3
Lactate dehydrogenase 1260 U/L


A) Acute toxic hepatitis

B) Alcoholic hepatitis

C) Cholangiocarcinoma

D) Chronic idiopathic cirrhosis

E) Common bile duct calculus

F) Congestive hepatitis

G) Gallstone pancreatitis

H) Gilbert's syndrome

I) Infectious hepatitis

J) Intravascular hemolysis

K) Primary biliary cirrhosis

12.) A 24-year-old third-year medical student comes to the student health clinic for
evaluation of jaundice which he noticed this morning. He has had no abdominal
pain, itching, or weight change. He takes no medications. He describes recent
anxiety over anticipation of the upcoming Step 2 examination. His temperature is
37 C (98.6 F), blood pressure is 120/80 mm Hg, pulse is 76/min, and respirations
are 12/min. Examination shows no abnormalities except for scleral icterus.

Laboratory studies show:

Hemoglobin 15 g/dL Serum
Leukocyte count 7000/mm3
Alkaline phosphatase 90U/L
Platelet count 240,000/mm3
Aspartate aminotransferase Serum (AST, GOT) 23 U/L
Bilirubin, total 3.5 mg/dL
Alanine aminotransferase Direct 0.7 mg/dL (ALT, GPT) 27 U/L


A) Acute toxic hepatitis

B) Alcoholic hepatitis

C) Cholangiocarcinoma

D) Chronic idiopathic cirrhosis

E) Common bile duct calculus

F) Congestive hepatitis

G) Gallstone pancreatitis

H) Gilbert's syndrome

I) Infectious hepatitis

J) Intravascular hemolysis

K) Primary biliary cirrhosis

13.) A 37-year-old woman with a 2-year history of hypertension comes for a follow-up
examination; her hypertension has worsened despite treatment with a low-sodium
diet and a ß-adrenergic blocking agent. She has a history of rheumatic fever and
Graves' disease treated with 131I. Her blood pressure is 160/106 mm Hg, and
pulse is 80/min. Serum studies show:

Na+ 135 mEq/L
Cl– 100 mEq/L
K+ 3.4 mEq/L
HCO3– 26 mEq/L
Urea nitrogen (BUN) 12 mg/dL
Creatinine 0.8 mg/dL

Urinalysis is within normal limits. Renal ultrasonography shows a left kidney with a
markedly irregular contour; it is 2.8 cm smaller than the right.


A) Adrenal cortex

B) Adrenal medulla

C) Aorta

D) Renal arteries

E) Renal glomeruli

F) Thyroid gland

14.) A 27-year-old woman comes to the physician because of muscle weakness and
cramps for 2 weeks. She has been taking a ß-adrenergic blocking agent for
hypertension for 2 years. She had chronic lymphocytic thyroiditis (Hashimoto's
disease) 1 year ago. Her blood pressure is 160/108 mm Hg, and pulse is 60/min.
Serum studies show:

Na+ 140 mEq/L
Cl– 110 mEq/L
K+ 2.2 mEq/L
HCO3– 30 mEq/L
Urea nitrogen (BUN) 20 mg/dL
Creatinine 1 mg/dL

Magnetic resonance angiography of the abdomen shows normal findings.

A) Adrenal cortex

B) Adrenal medulla

C) Aorta

D) Renal arteries

E) Renal glomeruli

F) Thyroid gland

15.) A 24-year-old man is hospitalized for treatment of a posterior dislocation of the
right knee sustained in a motorcycle collision. Six hours after closed reduction,
previously present distal pulses in his foot are absent, but the foot has remained
warm. Which of the following is the most appropriate next step in management?


A) Elevation of the limb and observation for 24hours

B) Nitroprusside therapy

C) Sympathetic block

D) Femoral arteriography

E) Embolectomy with a Fogarty catheter through aproximal arteriotomy

D) Inflammation of the origin of the plantar fascia





16.) A healthy 4-year-old girl is brought for a well-child examination. A grade 2/6
systolic ejection murmur is heard along the upper left sternal border. S2 is widely
split and does not vary with respiration. A soft mid-diastolic murmur is heard along
the lower left sternal border. Examination shows no other abnormalities. Which of
the following is the most likely diagnosis?


A) Aortic stenosis

B) Atrial septal defect

C) Coarctation of the aorta

D) Mitral valve prolapse

E) Patent ductus arteriosus

F) Pulmonary stenosis

G) Tetralogy of Fallot

H) Transposition of the great arteries

I) Ventricular septal defect

J) Normal heart



18.) A previously healthy 57-year-old woman comes to the physician because of three
episodes of blurred vision in the right eye over the past 3 weeks; each episode
lasts approximately 5minutes. Retinal examination shows a small refractile body at
The bifurcation of a retinal artery. The remainder of the examination shows no
abnormalities. Which of the following is the most appropriate next step in
diagnosis?

A) Cerebral angiography

B) Echocardiography

C) Electroencephalography

D) Duplex scan of the carotid arteries

E) MRI of the brain


19.) A 67-year-old woman comes to the physician for her first influenza virus
vaccination. She has a history of untreated hypertension. Her blood pressure is
160/100 mm Hg, and pulse is 100/min. Shortly after administration of the
influenza virus vaccine, she develops shortness of breath, hives, and angioedema.
Which of the following is most likely to have prevented this reaction?


A) Inquiry about an egg allergy

B) Heterophile agglutination test

C) Skin test with histamine reagent

D) ß-Adrenergic blocking agent therapy

E) Amantadine therapy

F) Insulin therapy

G) Rimantadine therapy



20.) A 7-year-old girl is brought to the physician because of a 2-day history of fever,
headache, sore throat, and swollen glands. She does not have a runny nose,
congestion, or cough. She has no allergies to medications. Her temperature is
38.6C (101.4 F), blood pressure is 100/60 mm Hg, pulse is 120/min, and
respirations are 16/min. Examination shows a swollen, erythematous oropharynx
With tonsillar exudates. The anterior cervical lymph nodes are enlarged and
tender. No other abnormalities are noted. Which of the following is the most likely
causal organism?

A) Adenovirus

B) Corynebacterium diphtheriae

C) Group A streptococcus

D) Haemophilus influenzae

E) Mycoplasma pneumoniae


21.) A 70-year-old nursing home resident is admitted to the hospital because of
progressive obtundation over the past 2 days. He has tachycardia, tachypnea, and
hypotension. Bilateral basilar crackles and an S3 gallop are heard on auscultation.
Examination shows jugular venous distention and peripheral edema. Swan-Ganz
catheterization shows a cardiac index of 1.8 L/min/m2 (N=2.5–4.2), a mean
Pulmonary capillary wedge pressure of 23 mm Hg (N=1–10), and markedly
Increased systemic vascular resistance. Which of the following is the most likely
diagnosis?

A) Cardiogenic shock

B) Hypovolemic shock

C) Neurogenic shock

D) Septic shock



22.) A 42-year-old man with alcoholism is brought to the emergency department by a
friend because of fever and progressive shortness of breath for 12 hours. His
friend reports that they were eating leftover chicken and drinking beer earlier in
the day when the patient suddenly choked and vomited. Six hours later, he
developed sweating, chills, and shortness of breath. He has not had any
subsequent nausea, vomiting, hematemesis, or abdominal pain. He has smoked
Two packs of cigarettes daily for 22 years and drinks 12 to 18 beers daily. He is in
moderate respiratory distress. His temperature is 39.3 C (102.8 F), blood pressure
is 90/60 mm Hg, pulse is 120/min, and respirations are 24/min. Examination
shows no jugular venous distention. Breath sounds are decreased halfway up the
left lung with increased dullness. Cardiac examination shows a normal S1 and S2;
no murmurs are heard. There is no abdominal tenderness. Bowel sounds are
hypoactive. X-ray films of the chest show a left pleural effusion and air in the
mediastinum. Thoracentesis is performed.
Laboratory studies show:
Leukocyte count 18,000/mm3
Segmented neutrophils 85%
Bands 10%
Lymphocytes 5%
Serum Protein 6 g/dL
Lactate dehydrogenase 200 U/L
Pleural fluid Leukocyte count 8000/mm3
Segmented neutrophils 98%
Monocytes 2%
Protein 4.2 g/dL
Amylase 140 U/L
Lactate dehydrogenase 180 U/L
Gram's stain
WBC present
Organisms none

Which of the following is the most likely diagnosis?

A) Congestive heart failure

B) Esophageal rupture

C) Pancreatitis

D) Pericarditis

E) Tuberculosis


23.) A 2-year-old girl with tricuspid atresia has increasing respiratory distress for 2
days. She has been recovering uneventfully from an operation 10 days ago to join
systemic venous return with pulmonary arterial circulation. Over the past 4 days,
she has been weaned off mechanical ventilation, started on oral feedings, and
is receiving chest physiotherapy for atelectasis. Her temperature is 37.4 C (99.3 F),
blood pressure is 98/64 mm Hg, pulse is 120/min, and respirations are 46/min.
Examination shows nasal flaring, grunting, and intercostal retractions. An x-ray
film of the chest shows large bilateral pleural effusions. Thoracentesis yields 300
mL of whitish-yellow fluid. The supernatant remains uniformly opaque on
centrifugation. Which of the following is the most likely cause of the pleural
effusions?

A) Chylothorax

B) Congestive heart failure

C) Empyema

D) Pulmonary embolism

E) Superior vena cava obstruction


24.) A 23-year-old man is brought to the physician by his mother because he has heard
a voice over the past month telling him to hurt himself. His mother says that her
son has no friends and is a lifelong loner; since graduating from high school, he
has been unable to hold a job. He admits to smoking marijuana occasionally and
drinking six beers weekly. Examination shows a poorly groomed man with poor
eye contact. He has a flat affect and limited facial expression. He says he has no
intention of harming himself or others. Which of the following is the most
appropriate next step in management?

A) Schedule a follow-up visit in 4 weeks

B) Prescribe oral risperidone and schedule a follow-up visit in 2 weeks

C) Admit him to the partial hospital program and prescribe oral lithium carbonate

D) Admit him to the psychiatric unit for detoxification

E) Admit him to the psychiatric unit and prescribe oral imipramine



25.) A 23-year-old man is brought to the physician by his mother because he has heard
a voice over the past month telling him to hurt himself. His mother says that her
son has no friends and is a lifelong loner; since graduating from high school, he
has been unable to hold a job. He admits to smoking marijuana occasionally and
Drinking six beers weekly. Examination shows a poorly groomed man with poor
Eye contact. He has a flat affect and limited facial expression. He says he
has no intention of harming himself or others. Which of the following is the most
appropriate next step in management?


A) Schedule a follow-up visit in 4 weeks

B) Prescribe oral risperidone and schedule a follow-up visit in 2 weeks

C) Admit him to the partial hospital program and prescribe oral lithium carbonate

D) Admit him to the psychiatric unit for detoxification

E) Admit him to the psychiatric unit and prescribe oral imipramine




26.) An 18-month-old boy is brought for a well-child examination. He was born at 37
weeks' gestation and weighed 2800 g (6 lb 3oz). There were no prenatal or
perinatal complications. Developmental history indicates that he is able to run and
walk up stairs while holding his mother's hand. He can hold a crayon but does not
Scribble spontaneously. He is at the 40th percentile for length and 50th percentile
for weight. Examination shows a crude pincer grasp. Which of the following is the
most appropriate assessment of fine and gross motor development?


Fine Motor Gross Motor
Development Development

A) Delayed normal

B) Normal delayed

C) Delayed delayed

D) Normal normal



27.) Five months after beginning fluoxetine to treat obsessive-compulsive disorder, a
19-year-old man states that he discontinued his medication 2 months ago because
he had begun to worry about taking his medication every day. His initial response
to the medication was good. His symptoms have now returned, and his morning
ritual of cleaning and grooming consumes so much time that his job is in jeopardy.
In addition to education about the nature of his disorder and its treatment, which
of the following is the most appropriate next step in management?


A) Tell the patient to schedule a return visit as needed

B) Offer to change the medication

C) Request that the patient's parents superviseadministration of medication

D) Monitor the patient's compliance by weekly blood tests

E) Begin a trial of cognitive-behavior therapy




28.) A 32-year-old woman comes to the physician because of a 3-month history of
increasing pain and stiffness in her wrists, hands, and ankles. During this period,
she also has had progressive fatigue and morning stiffness lasting 2 hours. She has
a 1-year history of rheumatoid arthritis treated with naproxen. Examination shows
redness, swelling, and warmth over the wrist, hand, and ankle joints bilaterally.
There are nontender subcutaneous nodules over the extensor surfaces of both
elbows. X-ray films of the hands show diffuse osteopenia and erosions over several
of the distal metacarpal bones. Which of the following is the most appropriate
pharmacotherapy?

A) Add oral cyclophosphamide

B) Add oral gold

C) Add oral methotrexate

D) Add oral penicillamine

E ) Switch to oral ibuprofen



28.) A previously healthy 62-year-old man comes to the physician because of a 2-month
history of progressive shortness of breath and a mild nonproductive cough. He
does not smoke. He worked in a foundry most of his adult life before retiring 2
years ago. Vital signs are within normal limits. Crackles are heard at both lung
Bases with no wheezes. Cardiac examination shows an accentuated P2. The
remainder of the examination shows no abnormalities. An x-ray film of the chest
shows prominent interstitial markings at the lung bases. Echocardiography shows
an ejection fraction of 55%. Pulmonary function testing is most likely to show
which of the following?

A) Decreased FEV1:FVC ratio

B) Decreased maximal inspiratory effort

C) Decreased total lung capacity

D) Increased forced vital capacity

E) Normal carbon monoxide diffusion capacity



29.) A 55-year-old man is admitted to the hospital because of progressive shortness of
breath for 10 days. He has a history of chronic venous stasis and deep venous
thrombosis. He has been treated with warfarin since he had several pulmonary
emboli 2 years ago; he takes no other medication. He has smoked two packs of
Cigarettes daily for 30 years. He weighs 109 kg (240 lb) and is 165 cm (65 in) tall.
Examination shows jugular venous distention. Echocardiography is most likely to
show which of the following?

A) Left ventricular dilation

B) Mitral stenosis

C) Pericardial thickening and tamponade

D) Right ventricular hypertrophy and dilation

E) Tricuspid stenosis


30.) A 32-year-old woman comes to the physician because of weakness of the lower
extremities for 2 days. Three years ago, she had pain and partial loss of vision of
the right eye; the vision returned to normal after 6 weeks. There is mild pallor of
the right optic disc. She has impaired tandem gait. Babinski's sign is present
bilaterally. There is mild spasticity of the lower extremities and mild weakness of
the iliopsoas and hamstring muscles. Serum creatine kinase activity is 50 U/L.
Which of the following is the most likely diagnosis?

A) Amyotrophic lateral sclerosis

B) Becker's muscular dystrophy

C) Diffuse sensorimotor peripheral neuropathy

D) Duchenne's muscular dystrophy

E) Hyperkalemic periodic paralysis

F) Hypokalemic periodic paralysis

G) Multiple sclerosis

H) Myasthenia gravis

I) Myasthenic (Lambert-Eaton) syndrome

J) Myotonic muscular dystrophy

K) Polymyositis

L) Pontine astrocytoma

M) Pontine infarction

N) Spinal cord tumor

O) Transverse myelitis



31.) A healthy 42-year-old man comes to the physician for a life insurance evaluation.
He smoked one-half pack of cigarettes daily for 20 years but quit 10 years ago. His
father died of a myocardial infarction at the age of 65 years. The patient weighs 93
kg (205lb) and is 178 cm (70 in) tall. His blood pressure is 160/110 mm Hg,
pulse is 96/min, and respirations are 16/min. Physical examination, ECG, and an x-
ray film of the chest show no abnormalities. Laboratory studies are within normal
limits except for a serum cholesterol level of 206 mg/dL. Which of the following is
the greatest risk factor for cerebral infarction in this patient?

A) Genetic profile

B) History of smoking

C) Hypercholesterolemia

D) Hypertension

E) Obesity


32.) A previously healthy 56-year-old woman comes to the physician because of
jaundice and dark urine for 3 weeks. She has a 1-year history of generalized
pruritus. She takes no medications. Examination shows jaundice and several
ecchymoses over the forearms and thighs. The liver and spleen are enlarged and
nontender. Laboratory studies show:

Prothrombin time 18 sec
Serum Protein Total 8.5 g/dL
Albumin 3.8 g/dL
Bilirubin Total 5 mg/dL
Direct 2 mg/dL
Alkaline phosphatase 150 U/L
Alanine aminotransferase (ALT, GPT) 45 U/L
Antimitochondrial antibody assay is strongly positive.

A CT scan of the abdomen shows hepatosplenomegaly.
Endoscopic retrograde cholangiopancreatography shows no abnormalities.

Because of her condition, this patient is at greatest risk for which of the following
deficiencies?


A) Niacin

B) Vitamin A

C) Vitamin B2 (riboflavin)

D) Vitamin B12 (cyanocobalamin)

E) Vitamin C


33.) An asymptomatic 52-year-old man comes for a
follow-up
examination 1 month after he passed renal calculi. He
has a history of renal
calculi 2 years ago. Serum uric acid and calcium
levels and urinary
oxalate excretion are within normal limits. Urinary
calcium excretion is
increased. In order to avoid recurrence of renal
calculi, which of the
following is the most appropriate pharmacotherapy for
this patient?

A
) Bicarbonate

B
) Calcium lactate

C
) Methenamine mandelate

D
) Probenecid

E
) Thiazide diuretic


34.

A 57-year-old woman is brought to the emergency
department because of
abdominal pain for 12 hours. Over the past 3 hours,
the pain has become
severe and generalized. Over the past month, she has
had mild upper
abdominal discomfort that is relieved by eating. She
has a history of
recurrent migraines treated with sumatriptan as
needed. Her temperature
is 38.2 C (100.8 F), blood pressure is 170/95 mm Hg,
and pulse is
110/min. Abdominal examination shows mild distention;
there is marked
rigidity with diffuse tenderness. Bowel sounds are
absent. Rectal
examination shows no abnormalities; test of the stool
for occult blood is
negative. Laboratory studies show:


Hematocrit 36%
Leukocyte count 16,500/mm3
Serum
Na+ 145 mEq/L
Cl– 106 mEq/L
K+ 3.8 mEq/L
HCO3– 19 mEq/L
Urea nitrogen (BUN) 32 mg/dL

Which of the following is the most appropriate next
step in diagnosis?

A
) X-ray films of the abdomen while supine and
standing

B
) Abdominal ultrasonography

C
) Upper gastrointestinal series with contrast

D
) HIDA scan

E
) Fiberoptic endoscopy of the upper
gastrointestinal tract



35. An 18-year-old man comes to the physician
because of itchy
lesions on his penis for 2 weeks and itching around
his wrists and ankles
for 1 week. He is sexually active and does not
consistently use condoms.
Examination shows scattered, crusted lesions on the
penis and no
significant inguinal lymphadenopathy. There are
excoriated papules on the
wrists and ankles, a few papules between the fingers,
and excoriations
along the belt line. Which of the following is the
most likely
diagnosis?

A
) Disseminated primary herpes simplex

B
) Eczema

C
) Scabies

D
) Secondary syphilis

E
) Vasculitis


36. An asymptomatic 62-year-old man comes for a
follow-up visit.
One month ago, he had acute cystitis treated with
ciprofloxacin. At his
initial visit, a urine culture grew Klebsiella
pneumoniae. He has a
2-year history of chronic prostatitis and has had four
episodes of
cystitis over the past year. His temperature is 36.9
C (98.4 F).
Examination, including prostate examination, shows no
abnormalities. Which of
the following is the most appropriate measure to
prevent recurrent
episodes of cystitis in this patient?

A
) Trimethoprim-sulfamethoxazole prophylaxis

B
) Insertion of a Foley catheter

C
) Total prostatectomy

D
) Transurethral prostatectomy

E
) No treatment is available


37. A 32-year-old woman comes for a routine health
maintenance
examination. Examination shows a bloody discharge
from the nipple of her
left breast. There is no palpable mass. On
questioning, she says that
she never noted the discharge and does not perform
breast
self-examinations. Which of the following is the most
likely diagnosis?

A
) Breast abscess

B
) Breast cyst

C
) Breast hematoma

D
) Fibroadenoma

E
) Fibrocystic changes of the breast

F
) Intraductal adenoma


38. A 35-year-old woman is brought to the physician
by her husband
because of increasing memory loss and involuntary
"dancing" movements
over the past 6 weeks. She has had difficulty
remembering things,
particularly recent events. She has had no
disturbances in sleep or appetite.
Her medical history is unremarkable. She does not
remember her mother
who died 25 years ago, but her father told her that
her mother had
similar symptoms several years before her death.
Neurologic examination
shows involuntary choreiform movements. She has a
childlike affect. She
describes her mood as good. On recall testing, she
can remember one out
of three objects after 3 minutes with distraction; she
cannot remember
the other objects with prompting. She cannot remember
the date of her
marriage or her previous address. The most likely
cause of these
symptoms is a lesion at which of the following
locations?

A
) Caudate nucleus

B
) Parietal lobe

C
) Prefrontal lobe

D
) Putamen

E
) Temporal lobe


39. A 67-year-old man comes to the physician
because of insomnia for
2 years. He goes to bed at 11:00 PM after taking a
bath but does not
fall asleep until midnight. He usually wakes up twice
each night: once
around 3:00 AM to void and again at 6:00 AM. He stays
in bed until his
alarm goes off at 7 AM. He is concerned because he
used to sleep 8
hours daily. He recently retired from his job as an
attorney. He takes
no medications. He drinks one beer with lunch 3 days
each week and one
glass of wine with dinner each evening. He walks 1
mile daily after
dinner. Physical examination shows no abnormalities.
His mood is
neutral, and cognition is intact. Urine toxicology
screening is negative.
Which of the following is the most likely cause of
this patient's
symptoms?

A
) Advanced sleep phase syndrome

B
) Alcohol abuse

C
) Delayed sleep phase syndrome

D
) Major depressive disorder

E
) Melatonin deficiency

F
) Poor sleep hygiene

G
) Normal aging


40. An asymptomatic 37-year-old woman comes for a
follow-up
examination 1 year after receiving a renal transplant.
Current medications
include felodipine, enalapril, cyclosporine,
pravastatin, and penicillin
G. She appears well. Her temperature is 36.8 C (98.2
F), blood
pressure is 160/95 mm Hg, pulse is 80/min, and
respirations are 12/min.
Examination shows no other abnormalities. Which of
the following
medications is the most likely cause of this patient's
hypertension?

A
) Cyclosporine

B
) Enalapril

C
) Felodipine

D
) Penicillin G

E
) Pravastatin


41. A 4080-g (9-lb) male newborn is delivered at
term to a
32-year-old woman, gravida 2, para 1. Apgar scores
are 8 and 9 at 1 and 5
minutes, respectively. Examination in the delivery
room shows fracture of
the right clavicle. Which of the following is the
most likely sequela
of this condition?

A
) Correction only with casting

B
) Correction only with physical therapy

C
) Left-hand dominance

D
) Permanent nerve damage

E
) Spontaneous healing without treatment


42. A 22-year-old man with schizoaffective disorder
is brought to
the emergency department 2 hours after the sudden
onset of neck and back
pain. The symptoms began after taking one of his
medications for his
psychiatric disorder. The dosage was increased
yesterday, but he does
not recall the names of either of his medications. He
has no other
history of serious illness. Physical examination
shows rigid contraction
of the neck and back muscles with arching. On mental
status
examination, he is alert and cooperative and hears a
faint voice that tells him to
wash windows. He is oriented to person, place, and
time. Which of the
following medications is most likely responsible for
this patient's
symptoms?

A
) Clozapine

B
) Haloperidol

C
) Lithium carbonate

D
) Trazodone

E
) Valproic acid

43. A 52-year-old woman, gravida 3, para 3, comes
to the physician
because of irregular vaginal bleeding over the past 2
months. She has
hypertension treated with enalapril and type 2
diabetes mellitus well
controlled with diet. Menopause occurred 2 years ago.
Her maternal aunt
had breast cancer at the age of 70 years. The patient
weighs 88 kg
(195 lb) and is 160 cm (63 in) tall. Examination
shows no other
abnormalities. An endometrial biopsy specimen shows
adenocarcinoma. Which of
the following is the most significant predisposing
factor for this
patient's endometrial cancer?

A
) Heredity

B
) Hypertension

C
) Parity

D
) Type 2 diabetes mellitus

E
) Weight


44. A 67-year-old man is brought to the emergency
department 2 hours
after the onset of weakness and double vision. He has
hypertension and
hyperlipidemia treated with metoprolol, captopril, and
atorvastatin.
His blood pressure is 190/106 mm Hg. Neurologic
examination shows
left-sided facial weakness including the forehead.
There is palsy of left
conjugate gaze, and the left eye fails to adduct on
right gaze.
Vertical eye movements are intact. Muscle strength is
3/5 in the right upper
and lower extremities. Deep tendon reflexes are
brisk, and Babinski's
sign is present on the right. Which of the following
is the most
likely location of this patient's lesion?

A
) Bilateral thalamic

B
) Left frontal

C
) Left pontine

D
) Right caudate

E
) Right midbrain


45. Six weeks after spontaneous drainage of an anal
abscess, a
32-year-old man has persistent blood-stained purulent
fluid on his
underwear. He has not had significant anal pain since
drainage of the boil.
Bowel movements are normal. Which of the following is
the most likely
diagnosis?

A
) Anal fissure

B
) Fistula in ano

C
) Pruritus ani

D
) Thrombosed external hemorrhoids

E
) Thrombosed internal hemorrhoids


46. A previously healthy 14-year-old girl is
brought to the
physician because of a 2-day history of fever and pain
and swelling of the
right knee. She remembers injuring the knee while
playing soccer last
week, but she was able to finish the game. She has no
history of rash or
joint pain. Her sister has inflammatory bowel
disease. The patient's
temperature is 39 C (102.2 F), blood pressure is
110/80 mm Hg, pulse is
95/min, and respirations are 20/min. Examination of
the right knee
shows swelling, tenderness, warmth, and erythema;
range of motion is
limited. Which of the following is the most
appropriate next step in
management?

A
) X-ray film of the right knee

B
) Gastrointestinal series with small-bowel

follow-through

C
) Nuclear scan of the right knee

D
) MRI of the right knee

E
) Antibiotic therapy

F
) Arthrocentesis




block 2

dolly123 - 11/07/06 17:58 #548028
Share
Views: 8566
Replies to This Discussion
Permalink Reply by usmle99 on March 14, 2009 at 11:10pm
1.

A 25-year-old man is brought to the emergency
department after being
discovered semiconscious and incoherent at home. On
arrival, he is
stuporous. His blood pressure is 105/70 mm Hg, pulse
is 80/min, and
respirations are 12/min. Examination shows cool, damp
skin. The pupils are
pinpoint and react sluggishly to light. Which of the
following is the
most likely substance taken?

A
) Alcohol

B
) Barbiturates

C
) Cocaine

D
) Heroin

E
) LSD



2. A 15-year-old girl is brought to the emergency
department by her
20-year-old sister because of a 1-week history of
fatigue, nausea, and
abdominal pain. Menarche was at the age of 12 years,
and her last
menstrual period was 3 weeks ago. She has not had
vaginal discharge. She
is sexually active, and she and her partner use
condoms inconsistently.
Her temperature is 37 C (98.6 F), blood pressure is
110/60 mm Hg, and
pulse is 95/min. Abdominal examination shows mild
bilateral lower
quadrant tenderness. Before a pelvic examination and
a pregnancy test can
be performed in this patient, consent must be obtained
from which of the
following?

A
) The court

B
) The patient

C
) The patient's parent

D
) The patient's sister

E
) No consent is necessary



3. A previously healthy 57-year-old man comes to
the physician
because of a nonpruritic rash over both legs for 1
week and a low-grade
fever for 2 days. He recently returned from a 2-week
canoe trip on a river
in Minnesota. His temperature is 37.2 C (99 F).
Examination shows a 4
x 6-cm, macular, dark pink, ovoid lesion on the right
posterior hip
with central clearing and a punctate eschar near the
center. There are
macular ring lesions with central clearing over the
medial tibia and
anterior thighs of the lower extremities. He has no
lymphadenopathy.
Which of the following is the most likely causal
organism?

A
) Borrelia burgdorferi

B
) Brucella melitensis

C
) Francisella tularensis

D
) Leptospira interrogans

E
) Rickettsia rickettsii


4. A 57-year-old woman comes to the physician
because of a 2-year
history of increasing menstrual flow. She has not had
hot flashes,
insomnia, or change in bowel or bladder function. Her
last menstrual period
was 2 weeks ago. Pelvic examination shows a
normal-appearing vulva,
vagina, and cervix. The uterus is consistent in size
with an 8-week
gestation. Bimanual examination shows a 4-cm, firm,
nontender left ovary. An
endometrial biopsy specimen shows atypical complex
endometrial
hyperplasia. Which of the following is the most likely
cause of this patient's
hyperplasia?

A
) Adrenal adenoma

B
) Brenner tumor

C
) Carcinoid tumor

D
) Granulosa cell tumor

E
) Hyperthecosis

F
) Islet cell tumor

G
) Sertoli-Leydig cell tumor


5. A 67-year-old woman comes to the physician
because of vaginal
bleeding for 10 days. She has been soaking one
sanitary pad daily.
Menopause was 10 years ago. Her last Pap smear 5
years ago showed normal
findings. Her temperature is 37 C (98.6 F), blood
pressure is 128/78 mm
Hg, pulse is 70/min, and respirations are 12/min.
Pelvic examination
shows a normal-appearing cervix and a small amount of
blood at the
cervical os. There is moderate thinning of the
vaginal mucosa. The uterus
is irregular and consistent in size with a 10-week
gestation. There
are no adnexal masses. Rectovaginal examination shows
no abnormalities.
Test of the stool for occult blood is negative. Which
of the following
is the most appropriate next step in diagnosis?

A
) Transvaginal ultrasonography of the pelvis

B
) Colposcopic-guided biopsy

C
) Endometrial biopsy

D
) Endometrial ablation

E
) Culdocentesis


6. During a routine examination, a 32-year-old man
has a blood
pressure of 120/80 mm Hg. He is concerned because his
father, grandfather,
and two uncles have hypertension. He works as a
systems programmer for
a large computer company and frequently has to meet
tight deadlines.
He has smoked one pack of cigarettes daily for 10
years. He is 4.5 kg
(10 lb) overweight and drinks three cups of coffee
daily. Which of the
following measures is most likely to reduce this
patient's risk for
hypertension over the next 5 years?

A
) Increase intake of dietary fiber

B
) Restrict caffeine

C
) Stress management

D
) Weight loss


7. A previously healthy 16-year-old boy comes to
the physician
because of persistent pain in his left testicle for 24
hours. He has not
had any penile discharge. There is no history of
trauma, but he plays
soccer every day. Two months ago, he had sexual
intercourse for the
first time, and he used a condom. Examination shows
an edematous,
erythematous, exquisitely tender left scrotum that is
lower than the right.
Elevating the left testicle relieves the pain. The
cremasteric reflex is
present. Urinalysis shows 10 leukocytes/hpf and 1+
leukocyte esterase.
A technetium 99m scan shows increased uptake in the
left testicle.
Which of the following is the most likely cause of the
pain?

A
) Cystitis

B
) Epididymitis

C
) Spermatocele

D
) Testicular torsion

E
) Testicular tumor


8. A 47-year-old woman comes to the physician
because of persistent
nonproductive cough for 6 weeks. She has not had
fever or weight loss.
She has hypertension treated with enalapril for the
past 3 months. She
does not smoke. There is no history of lung disease.
She weighs 54 kg
(120 lb) and is 163 cm (64 in) tall. Her temperature
is 37 C (98.6 F),
blood pressure is 130/80 mm Hg, pulse is 70/min, and
respirations are
12/min. Examination and an x-ray film of the chest
show no
abnormalities. Which of the following is the most
likely mechanism of this
patient's cough?

A
) Decreased plasma renin activity

B
) Decreased serum angiotensin II levels

C
) Increased serum angiotensin I levels

D
) Increased serum bradykinin levels

E
) Increased serum histamine levels


9. A 57-year-old man has been hospitalized for 2
days for treatment
of unstable angina pectoris. He is currently
receiving intravenous
heparin and undergoing evaluation for coronary artery
bypass grafting.
His blood pressure is 160/90 mm Hg, pulse is 88/min,
and respirations are
16/min. Laboratory studies show:

Platelet count 90,000/mm3
Prothrombin time 12 sec (INR=1.1)
Partial thromboplastin time 35 sec

Which of the following is the most likely cause of
these findings?

A
) Excessive platelet destruction

B
) Factor VIII deficiency

C
) Inadequate platelet production

D
) Uncontrolled activation of coagulation and
fibrinolytic cascades

E
) Vitamin K deficiency


10. A study is conducted to assess the
effectiveness of a new blood
test for early detection of prostate cancer. Ten
thousand healthy men
over the age of 50 years are randomly assigned to
receive either annual
rectal examination or annual screening with the new
blood test. After
5 years, results show that of the 50 men in the blood
test group that
were diagnosed with prostate cancer, 40 were living 2
years after the
diagnosis was made. In comparison, only 15 out of 45
men in the rectal
examination group survived 2 years after being
diagnosed with prostate
cancer. Researchers conclude that the blood test
increases survival
compared with rectal examination. Which of the
following potential flaws
is most likely to invalidate this conclusion?

A
) Age of the patients

B
) Diagnostic bias

C
) Lead time bias

D
) Recall bias

E
) Type II error


11.

A 62-year-old man comes to the emergency department
because of
progressive shortness of breath for 3 days. He has
not had chest pain,
orthopnea, or paroxysmal nocturnal dyspnea. He
completed chemotherapy for
small cell carcinoma of the lung 10 months ago. He
has a history of twice
nightly nocturia that has resolved over the past 3
days. He smoked two
packs of cigarettes daily for 30 years but quit 1 year
ago. His blood
pressure is 96/60 mm Hg, and pulse is 116/min. There
is jugular venous
distention to the angle of the jaw. The lungs are
clear to
auscultation. Cardiac examination shows distant heart
sounds, an S1 and S2, and no
gallops or rubs. The liver has a span of 12 cm and is
tender. There
is no pedal edema. Laboratory studies show:


Hemoglobin 10 g/dL
Serum
Na+ 135 mEq/L
Cl– 110 mEq/L
K+ 4.2 mEq/L
HCO3– 22 mEq/L
Urea nitrogen (BUN) 40 mg/dL
Creatinine 1.6 mg/dL

An ECG shows diminished amplitude of the QRS
complexes. An x-ray film
of the chest shows clear lung fields with an enlarged
cardiac
silhouette. Which of the following findings is most
likely to be accentuated?

A
) Cardiac output

B
) Fall in systolic arterial pressure with
inspiration

C
) Left ventricular end-diastolic pressure

D
) Mitral regurgitation

E
) Ventricular septal wall motion



12. A 35-year-old woman comes to the physician
because of two
12-hour episodes of dizziness over the past 3 months.
During episodes, she
experiences the acute onset of rotatory vertigo and
imbalance, decreased
hearing, tinnitus, a sense of fullness of the right
ear, and vomiting.
Examination shows a mild hearing loss of the right
ear. Which of the
following is the most likely diagnosis?

A
) Acoustic neuroma

B
) Benign positional vertigo

C
) Brain stem transient ischemic attacks

D
) Meniere's disease

E
) Viral labyrinthitis


3. An obese 33-year-old woman has had four 12-hour
episodes of
severe, sharp, penetrating pain in the right upper
quadrant of the abdomen
associated with vomiting but no fever. She has no
diarrhea, dysuria, or
jaundice and is asymptomatic between episodes. There
is slight
tenderness to deep palpation in the right upper
quadrant. Which of the
following is the most appropriate next step in
diagnosis?

A
) Supine and erect x-ray films of the abdomen

B
) Upper gastrointestinal series

C
) Ultrasonography of the upper abdomen

D
) CT scan of the abdomen

E
) HIDA scan of the biliary tract

14. An otherwise healthy 19-year-old woman comes to
the physician
because of a 3-year history of intermittent facial
blemishes. She drinks
wine occasionally on weekends. She takes no
medications. Examination
shows multiple 1- to 2-mm red and white papules and
larger red nodules
on the forehead and cheeks. Which of the following is
the most
appropriate initial pharmacotherapy?

A
) Oral isotretinoin

B
) Systemic corticosteroids

C
) Topical benzoyl peroxide

D
) Topical corticosteroids

E
) Topical metronidazole



15. A previously healthy 67-year-old man comes to
the physician
because of a 4-month history of hand weakness,
intermittent tingling of the
small fingers of his hands, and mild neck pain.
Examination shows
wasting, weakness, and fasciculations of the
interossei muscles. Sensation
is decreased to pinprick and vibration in the small
fingers of each
hand. Triceps tendon reflexes are decreased. Which
of the following is
the most likely diagnosis?

A
) Amyotrophic lateral sclerosis

B
) Cervical spondylosis

C
) Multiple sclerosis

D
) Myasthenia gravis

E
) Myasthenic (Lambert-Eaton) syndrome

F
) Myotonic muscular dystrophy

G
) Polymyalgia rheumatica

H
) Polymyositis

I
) Progressive neuropathic (peroneal) muscular

atrophY



16. A 19-year-old college student comes to student
health services
because of constant worrying since starting his
freshman year 6 months
ago. He attends college several hundred miles away
from his hometown.
He reports feeling constantly scrutinized by other
students and
professors. He feels embarrassed and anxious in class
and is in constant fear
of blushing. He has started to skip his classes
because of his anxiety
and worries that he will not be able to complete the
school year. He
drinks alcohol occasionally because it helps him
overcome his fear of
being embarrassed in front of others. He does not use
illicit drugs.
Physical examination shows no abnormalities. On
mental status
examination, he is mildly anxious. He exhibits fair
eye contact and shifts
uncomfortably in his chair. Laboratory studies are
within normal limits.
Which of the following is the most appropriate
pharmacotherapy for this
patient?

A
) Disulfiram

B
) Donepezil

C
) Fluoxetine

D
) Haloperidol

E
) Methylphenidate

F
) Valproic acid


17.

A 42-year-old woman comes for a follow-up examination.
Two weeks ago,
her blood pressure was 152/94 mm Hg during a routine
visit. Her blood
pressure today is 150/94 mm Hg, pulse is 76/min, and
respirations are
14/min. Examination shows no other abnormalities.
Serum studies show:


Na+ 142 mEq/L
Cl– 105 mEq/L
K+ 4 mEq/L
HCO3– 26 mEq/L
Urea nitrogen (BUN) 12 mg/dL
Glucose 101 mg/dL
Creatinine 0.8 mg/dL

An ECG shows no abnormalities. Which of the following
is the most
appropriate next step in management?

A
) Measurement of plasma renin activity

B
) Serum lipid studies

C
) 24-Hour urine collection for measurement of
metanephrine level

D
) Echocardiography

E
) Captopril renal scan


18. A 6-year-old boy with cystic fibrosis is
brought to the
physician by his mother because his skin has been cool
and clammy for 30
minutes. Earlier in the day, he had been playing
outdoors, and the
temperature was 99 F. When returning indoors, he was
thirsty and restless. His
blood pressure is 70/40 mm Hg, and pulse is 120/min.
Examination shows
dry mucous membranes. Serum sodium level is 128
mEq/L, and serum
chloride level is 87 mEq/L. Which of the following is
the most likely
explanation for these findings?

A
) Excessive sweat electrolyte level

B
) Excessive sweat volume

C
) Excessive urinary output

D
) Excessive vasopressor secretion

E
) Inadequate sweat production



19.

A 27-year-old primigravid woman at 29 weeks' gestation
comes to the
emergency department because of a 24-hour history of
increasingly severe
right-sided abdominal pain and no appetite. She has
vomited twice over
the past 4 hours. She has not had vaginal bleeding.
Her temperature
is 38.2 C (100.8 F). Examination shows exquisite
tenderness of the
right lateral flank and the fundus. There are no
peritoneal signs. Bowel
sounds are absent. The fetal heart rate is 144/min.
Laboratory
studies show:


Hematocrit 37%
Leukocyte count 16,000/mm3
Serum
Total bilirubin 1.1 mg/dL
Amylase 32 U/L
Lactate dehydrogenase 110 U/L
Urine WBC 3–5/hpf

Which of the following is the most likely diagnosis?

A
) Abruptio placentae

B
) Appendicitis

C
) Cholelithiasis

D
) Colitis

E
) Pyelonephritis


20. A 32-year-old woman has had a lump in her neck
for 6 months.
She has a 1.2-cm solitary left thyroid nodule.
Fine-needle aspiration
cytology of the mass is consistent with a low-grade
papillary malignancy.
Which of the following is the most appropriate next
step in management?

A
) Thyroid scan

B
) 131I therapy

C
) Propylthiouracil therapy

D
) Thyroxine therapy

E
) Left thyroid lobectomy


For each patient with vaginal bleeding, select the
most likely
diagnosis.


A
) Bacterial vaginosis

B
) Precocious puberty

C
) Urinary tract infection

D
) Vaginal foreign body

E
) Vaginal laceration

F
) von Willebrand's disease

21. A 12-year-old girl is brought to the physician
by her mother
because of heavy vaginal bleeding since her first
menstrual period began 1
week ago. She has to change sanitary pads every 2
hours, and her
mother is concerned that this is not normal. Her
blood pressure is 80/60 mm
Hg, and pulse is 110/min. Breast and axillary and
pubic hair
development are Tanner stage 4. Pelvic examination
shows normal external
genitalia, a small normal-appearing cervix, and a
small uterus. Her
hemoglobin level is 7 g/dL.



22. A 7-year-old girl is brought to the physician
by her mother 4
hours after she noticed that her daughter had blood on
her underwear.
She has no signs of sexual development. Examination
shows normal
external genitalia. There is a foul-smelling
discharge.


23.

A 10-year-old girl is brought to the physician because
of temperatures
to 40 C (104 F) and headaches for 1 week. Her mother
says that fever
and chills occur every other day and typically last
for several hours.
She appears ill and is lethargic during febrile
episodes. She and her
family returned from a trip to West Africa 2 weeks
ago. She currently
appears ill. Her temperature is 40 C (104 F), blood
pressure is 94/64
mm Hg, pulse is 146/min, and respirations are 20/min.
Examination shows
mild scleral icterus and conjunctival pallor. The
liver edge is
palpated 3 cm below the right costal margin, and the
spleen tip is palpated 3
cm below the left costal margin. Laboratory studies
show:


Hemoglobin 8 g/dL
Leukocyte count 6400/mm3
Segmented neutrophils 46%
Eosinophils 5%
Lymphocytes 40%
Monocytes 9%
Serum
Na+ 132 mEq/L
Cl– 98 mEq/L
K+ 4.2 mEq/L
HCO3– 16 mEq/L
Urea nitrogen (BUN) 21 mg/dL
Bilirubin
Total 5.2 mg/dL
Direct 0.8 mg/dL
Aspartate aminotransferase (AST, GOT) 64 U/L
Alanine aminotransferase (ALT, GPT) 98 U/L
Urine
Color tea
Blood strongly positive
RBC occasional

Which of the following measures is most likely to have
prevented this
condition?

A
) Hepatitis A vaccine

B
) Typhoid vaccine

C
) Oral isoniazid prophylaxis

D
) Oral mefloquine prophylaxis

E
) Oral trimethoprim-sulfamethoxazole prophylaxis

F
) Intramuscular immune globulin



24. A 6-month-old boy is brought to the physician
because of
seizures over the past week. His parents note that
during this period, he has
had 10 to 20 episodes of throwing out his arms for 1
to 2 seconds then
crying as if he were afraid of something. He was born
at term
following an uncomplicated pregnancy and delivery.
Development is appropriate
for age. Two days ago, treatment with amoxicillin was
begun for otitis
media. He appears pale and apathetic. Examination
shows multiple
white patches over the skin that become more distinct
with Wood's lamp
examination. Neurologic examination shows no
abnormalities. An MRI of the
brain is shown. Which of the following is the most
likely diagnosis?

A
) Congenital cytomegalovirus encephalopathy

B
) Congenital toxoplasmosis encephalopathy

C
) Hypoxic-ischemic encephalopathy

D
) Neurofibromatosis

E
) Tuberous sclerosis


25.

An ELISA is used to detect Chlamydia trachomatis
infection in patients
seen at a family planning clinic. In the first 500
patients, cultures
are also done to check the accuracy of the ELISA. The
results are
listed below:


Chlamydia culture
+

-

ELISA
+

38
5
-

2


455


Which of the following is the positive predictive
value of the ELISA?

A
) 2/40

B
) 5/43

C
) 38/40

D
) 38/43

E
) 38/493



26. A 27-year-old woman is brought to the emergency
department by
her mother who found her comatose 30 minutes ago. Her
mother says that
her daughter had been having lower abdominal pain and
vaginal bleeding
over the past week. The patient had an ectopic
pregnancy 2 years ago
and was also treated with doxycycline for pelvic
inflammatory disease at
that time. Her blood pressure is 40/20 mm Hg, pulse
is 160/min, and
respirations are 24/min. The abdomen is distended and
rigid with
decreased bowel sounds. Hemoglobin level is 4.2 g/dL,
and leukocyte count is
12,500/mm3. Culdocentesis is positive. Which of the
following is the
most appropriate next step in management?

A
) Bromocriptine therapy

B
) Clomiphene therapy

C
) Conjugated estrogen therapy

D
) Ergot derivative therapy

E
) Hysteroscopy

F
) Laparoscopy

G
) Dilatation and curettage

H
) Endometrial ablation

I
) Exploratory laparotomy

J
) Total abdominal hysterectomy



27. A 43-year-old man comes to the emergency
department because of
fever, chills, malaise, cough, and pleuritic chest
pain for 3 days; the
cough is productive of foul-smelling, purulent sputum.
He has
alcoholism. His temperature is 39.2 C (102.6 F),
blood pressure is 110/70 mm
Hg, pulse is 120/min, and respirations are 14/min.
Hemoglobin level is
15 g/dL, and leukocyte count is 25,000/mm3. An ECG
shows normal
findings. An x-ray film of the chest shows a 4-cm
cavity in the superior
segment of the right lower lobe and an air-fluid
level. While awaiting
results of sputum culture, which of the following is
the most appropriate
next step in management?

A
) Observation

B
) Broad-spectrum antibiotic therapy

C
) Isoniazid therapy

D
) Intravenous amphotericin B therapy

E
) Tube thoracostomy


28.

A 12-year-old girl with type 1 diabetes mellitus is
brought to the
physician because of shortness of breath and fatigue
for 1 day. Since
menarche began 4 months ago, she has had one episode
of diabetic
ketoacidosis per month; prior to that she had been
stable. Her blood pressure is
110/70 mm Hg, pulse is 140/min, and respirations are
36/min. She
appears to be moderately dehydrated. Laboratory
studies show:


Serum
Na+ 132 mEq/L
Cl– 90 mEq/L
K+ 5.9 mEq/L
HCO3– 6 mEq/L
Urea nitrogen (BUN) 48 mg/dL
Glucose 600 mg/dL
Creatinine 2.8 mg/dL
Urine
Glucose 4+
Ketones 3+
Protein 1+

Which of the following is the most likely underlying
cause of this
patient's respiratory disorder?

A
) Acute interstitial nephritis

B
) Acute renal failure

C
) Diabetic nephropathy

D
) Hyperinsulinemia

E
) Increased plasma ketone level


29. A 28-year-old woman develops fatigue and
orthopnea 3 weeks after
the uncomplicated delivery of her first child. Her
blood pressure is
115/78 mm Hg, and pulse is 112/min. Crackles are
heard at both lung
bases. The point of maximal impulse is laterally
displaced and diffuse;
an S3 is present. There is 2+ pedal edema. An x-ray
film of the chest
shows vascular cephalization of pulmonary vasculature
and Kerley B lines.
Which of the following is the most appropriate next
step in management?

A
) Measurement of serum creatine kinase and lactate
dehydrogenase
activities

B
) Echocardiography

C
) Dipyridamole thallium scan

D
) Ventilation-perfusion lung scans

E
) Myocardial biopsy


30. A previously healthy 22-year-old woman comes to
the physician
because of a paroxysmal cough for 2 weeks. She works
as an oncology
nurse. Her temperature is 37 C (98.6 F). Examination
shows no
abnormalities except for intermittent coughing spells
during the examination. An
x-ray film of the chest is normal. Cultures of
nasopharyngeal
secretions grow Bordetella pertussis. Which of the
following is the most
appropriate pharmacotherapy for this patient?

A
) Amoxicillin

B
) Amoxicillin-clavulanate

C
) Cefprozil

D
) Erythromycin

E
) Penicillin G


31. A 5-year-old girl is brought to the physician
because of a 2-day
history of temperatures to 39.5 C (103.1 F) and pain
in the right side.
She has had two episodes of vomiting during this
period but no diarrhea
or symptoms of upper respiratory tract infection. She
has
vesicoureteral reflux and a history of recurrent
urinary tract infections. She
appears ill. Her temperature is 39.8 C (103.6 F),
blood pressure is
110/60 mm Hg, pulse is 150/min, and respirations are
25/min. Examination
shows right-sided costovertebral angle tenderness. In
addition to
obtaining urine cultures, which of the following is
the most appropriate next
step in management?

A
) Intravenous pyelography

B
) Renal ultrasonography

C
) Voiding cystourethrography

D
) Intramuscular antibiotic therapy and
reexamination in 24 hours

E
) Intravenous antibiotic therapy


32. A 10-year-old boy is brought to the physician
because of
weakness and decreased appetite for 3 months. He
weighs 30 kg (66 lb) and is
142 cm (56 in) tall; he has had a 2.3-kg (5-lb) weight
loss since his
last examination 6 months ago. He appears thin. Deep
tendon reflexes
are brisk. Chvostek's sign is positive. His serum
calcium level is 6.5
mg/dL, and serum intact parathyroid hormone level is
190 pg/mL
(N=9–65). In addition to calcium supplementation,
which of the following is
the most appropriate next step in management?

A
) Oral phosphate supplementation

B
) Oral vitamin D supplementation

C
) Intranasal calcitonin therapy

D
) Parenteral bisphosphate therapy

E
) Parenteral parathyroid hormone therapy

Permalink Reply by usmle99 on March 14, 2009 at 11:11pm
33. A healthy 8-year-old girl is brought to the
physician in July
for a well-child examination. Her mother says that
her daughter is
spending the summer at a nearby lake. Over the past
month, she has had two
episodes of painful sunburn despite her mother's
efforts, including SPF
25 sunblock just before she goes swimming and urging
her to wear a hat
and long-sleeved garments. The child takes no
medications. She has
blond hair, blue eyes, and a fair complexion. The
mother seeks advice
about preventing further sun damage to her child's
skin. Which of the
following is the most appropriate recommendation?

A
) Prohibit swimming on cloudless days

B
) Apply the sunblock lotion 45 minutes before
swimming

C
) Change to a higher-level SPF lotion

D
) Apply Burrow's solution compresses after each
overexposure

E
) Daily use of antioxidant vitamin supplement

F
) Early treatment of any sun overexposure with
topical
corticosteroids





34. Two days after admission to the hospital
because of a 3-day
history of slurred speech, double vision, and
dysphagia, a 24-year-old
woman becomes quadriplegic and requires intubation and
mechanical
ventilation. Her medical history is unremarkable.
One week ago, she attended a
family picnic; several of her family members have had
abdominal cramps
and diarrhea since the picnic. Her temperature is 37
C (98.6 F), blood
pressure is 120/80 mm Hg, and pulse is 120/min.
Examination shows dry
mucous membranes, large unreactive pupils,
ophthalmoplegia, and
profound facial weakness. There is areflexia,
quadriplegia, and no movement
of the palate and tongue. Sensation is normal.
Babinski's sign is
absent. Which of the following is the most
appropriate pharmacotherapy?

A
) Antitoxin

B
) Azathioprine

C
) Interferon

D
) Pyridostigmine

E
) Riluzole



35. A 2325-g (5 lb 2 oz) male newborn is delivered
at 33 weeks'
gestation; Apgar scores are 7 and 8 at 1 and 5
minutes, respectively. The
13-year-old mother had no prenatal care and did not
know how much
weight she gained. During the pregnancy, the mother
smoked marijuana and
took over-the-counter vitamins occasionally; she did
not drink alcohol
and had no illness except for an upper respiratory
tract infection 4
months ago. She did not know she was pregnant until 2
weeks ago; her
family is unaware of her condition. She has had one
sexual partner. During
the hospital stay, the newborn and his mother have no
complications.
The newborn is at greatest risk for morbidity and
mortality from which
of the following?

A
) Child abuse

B
) Congenital syphilis

C
) Hypocalcemia

D
) Lead poisoning

E
) Seizures


36. A 2-month-old girl is brought to the physician
because of a
2-week history of progressive difficulty breathing and
poor feeding. She
has had rapid and labored breathing and sweating
during feedings. There
is no history of fever or viral illness. Her
temperature is 37 C (98.6
F), blood pressure is 80/60 mm Hg, pulse is 130/min,
and respirations
are 40/min. Bilateral crackles are heard at both lung
bases. A grade
4/6 holosystolic murmur is heard along the left
sternal border; the
precordium is hyperdynamic. The liver edge is
palpated 4 cm below the
right costal margin. An x-ray film of the chest shows
cardiomegaly and
pulmonary congestion. Which of the following is the
most likely
underlying mechanism for this child's condition?

A
) Decreased systemic vascular resistance

B
) Increased pulmonary vascular resistance

C
) Increased systemic vascular resistance

D
) Intracardiac left-to-right shunt

E
) Intracardiac right-to-left shunt



37. A 37-year-old man comes to the physician 6
months prior to
traveling to sub-Saharan Africa for 1 year. He has no
history of hepatitis
and has no high-risk behavior for hepatitis B. Which
of the following
is the most appropriate recommendation to prevent
hepatitis during and
after his trip?

A
) Immune globulin

B
) Hepatitis B immune globulin (HBIG) only

C
) Hepatitis B vaccine series only

D
) Hepatitis B vaccine series and HBIG

E
) Hepatitis B vaccine series and hepatitis A
vaccine



38.

A 67-year-old man comes to the physician because of a
2-month history
of progressive shortness of breath. He has had a
4.5-kg (10-lb) weight
loss over the past 4 months. He has not had chest
pain. He has
congestive heart failure treated with furosemide,
digoxin, and enalapril. He
has smoked two packs of cigarettes daily for 30 years.
He appears
alert and is in no acute distress. His temperature is
37.2 C (99 F), blood
pressure is 140/85 mm Hg, pulse is 84/min, and
respirations are 18/min.
Examination shows no jugular venous distention. There
is dullness to
percussion, and breath sounds are decreased at the
left base. C
Reply
#5
Exam Section 3

1. A 30-year-old woman has had progressive fatigue and muscle stiffness over the past 6 months. She is the primary caretaker for her three children and her elderly disabled parents and has had difficulty sleeping because of the needs of her children. She says that she has to stay motivated because of her responsibilities and has had no difficulty concentrating. She has not had suicidal ideation. She has had an increased appetite and has had a 14-kg (30-Ib) weight gain over the past year. She has a previous history of binge-eating and purging by vomiting. She has no history of psychoactive substance abuse or dependence, cold or heat intolerance, or changes in bowel
or urinary function. Range of motion of the upper extremities is limited by pain. There is no evidence of muscle atrophy. She has good eye contact. She describes her mood as down and stressed. Her affect is dysphoric and intermittently tearful. Which of the following is
the most appropriate pharmacotherapy?

O A) Bupropion
O B) Diazepam
O C) Fluoxetine
O D) Imipramine
0 E) Phenelzine


CCCCCCCCCC
This pt. has atypical depression, although it reponses well to MAOI but it also response well to SSRI, therefore, SSRI is prefered than MAOI because of less side effects.



2. A 13-year-old boy is brought to the physician by his parents because they are concerned about his behavior since starting eighth grade 5 months ago. They have noticed that he no longer seems interested in having dinner with the family and spends most of his time alone in his room playing loud music. He has quit his clarinet lessons. His grades have remained about the same, and they believe his peer relationships are unchanged. He has never had problems with law enforcement or school officials. When interviewed alone, the patient insists that everything is fine and says that he wishes his parents would stop treating him like a child. Examination, including mental
status, shows no abnormalities. Which of the following is the most likely explanation for this patient's behavior changes?
Q A) Conduct disorder
p B) Major depressive disorder
O C) Oppositional defiant disorder
p D) Substance abuse
0 E) Age-appropriate behavior

EEEEEEEEEEEEEEEEEE


The response options for the next two items are the same. You will be required to select one answer for each item in the set.
For each patient with a fetus or newborn with abnormalities, select the maternal condition most likely associated with these findings.
A) Alcohol use
B) Amphetamine use
C) Cigarette smoking
D) Cocaine use
E) Cystic fibrosis carrier state p F) Folic acid deficiency
G) Gametic nondisjunction
H) Heroin use
I) Iodine deficiency
J) Marijuana use
K) Parvovirus B19 infection
L) Rh sensitization
M) Rubella infection
N) Tay-Sachs carrier state

3. A 32-year-old woman, gravida 4, para 3, at 14 weeks' gestation comes for her first prenatal visit. Her previous pregnancy complications include concerns of fetal growth restriction and close surveillance by her physician. Her 4-year-old son with her today is small and learning delayed, and has been evaluated for microcephaly, hypertension, and presence of the hypoplastic philtrum.
For each patient with a fetus or newborn with abnormalities, select the maternal condition most likely associated with these findings.


AAAAAAAAAAAAAAAAAAA

A) Alcohol use
B) Amphetamine use
C) Cigarette smoking
D) Cocaine use
E) Cystic fibrosis carrier state p
F) Folic acid deficiency
G) Gametic nondisjunction
H) Heroin use
I) Iodine deficiency
J) Marijuana use
K) Parvovirus B19 infection
L) Rh sensitization
M) Rubella infection
N) Tay-Sachs carrier state


4. A 28-year-old woman, gravida 4. para 3, at 8 weeks' gestation comes for her first prenatal visit. She had a child who died shortly after birth of complications of bowel perforation. Her two other children are healthy. She and the father of her children are of Northern European extraction. There is no family history of neonatal death. She is concerned about recurrence.

EEEEEEEEEEEEEEEEEEE



5. A 62-year-old man comes to the physician for a follow-up examination. He has a 6-month history of increasingly severe, dull back pain and a 14-kg (30-Ib) weight loss despite a normal appetite. Recent x-ray films of the thoracolumbar spine showed no abnormalities. He has no history of trauma or serious illness. He takes no medications. He is 173 cm (5 ft 8 in) tall and weighs 91 kg (200 lb)-. BMI is 30 kglm2. Examination shows no other abnormalities. Test of the stool for occult blood is negative. A complete blood count, serum electrolyte levels, and urinalysis are within the reference range. Which of the following is the most appropriate next step in diagnosis?

O A) Measurement of serum CA 19-9 level
O B) Measurement of serum carcinoembryonic antigen (CEA) level
O C) CT scan of the abdomen
O D) Laparoscopy
O E) Upperendoscopy
O F) Mesenteric angiography

CCCCCCCCCCCCCCCCCCC



6. A 32-year-old woman comes to the physician because of a 5-year history of intermittent palpitations and vague nonradiating left-sided chest discomfort. Her symptoms usually last a few seconds and are not related to exertion. She is currently asymptomatic. Her pulse is
78/min, and blood pressure is 118168 mm Hg with no orthostatic changes. A midsystolic click and systolic murmur are heard best at the apex without radiation. The remainder of the examination shows no abnormalities. Which of the following is the most likely cause of
these findings?

O A) Aortic insufficiency
O B) Aortic stenosis
O C) Atrial septal defect
O D) Hypertrophic obstructive cardiomyopathy
O E) Mitral prolapse
O F) Mitral stenosis
O G) Pulmonic insufficiency
O H) Pulmonic stenosis
O I) Tricuspid insufficiency
O J) Tricuspid stenosis
0 K) Ventricular septal defect

EEEEEEEEEEEEEEEEEEE


7. A 67-year-old man is brought to the emergency department by his wife because of a 3-day history of progressive sleepiness. Over the past year, he has had intermittent night sweats and a 9-kg (20-Ib) weight loss with no change in appetite. He has a 2-year history of a nonproductive cough treated with inhaled nasal corticosteroid therapy. He has smoked two packs of cigarettes daily for 40 years and does not drink alcohol. He is somnolent and oriented to person but not to place or time. His temperature is 37.1 °C (98.8°F), pulse is 96/min, respirations are 16/min, and blood pressure is 120/85 mm Hg. Examination shows a supple neck with fullness in the left supraclavicular fossa. Breath sounds are decreased on the left. There is dullness to percussion at the left lung base. Neurologic examination shows no focal findings. Pulse oximetry on room air shows an oxygen saturation of 90%. A complete blood count is within the reference range, and serum calcium level is 14 mg/dL. Urinalysis shows a sodium level less than 5 mmol/L and a fractional sodium
excretion less than 1%. An x-ray film of the chest shows a large left pleural effusion. The most appropriate next step in management is administration of which of the following?

O A) Intravenous bisphosphonate
O B) Intravenous 5% dextrose in 0.45% saline
O C) Intravenous 5% dextrose in water
O D) Intravenous 0.9% saline
0 E) Oral loop diuretic

DDDDDDDDDDDDDDDDDD


8. A previously healthy 47-year-old man is brought to the emergency department after collapsing at work. His coworkers report that he had the sudden onset of a severe generalized headache before he lost consciousness. On arrival, he is lethargic and in severe distress. A lumbar puncture is performed. Cerebrospinal fluid analysis shows xanthochromia and an erythrocyte count of 40,0001mm3 in tubes 1 and 4. His wife and three children are at the hospital, and his parents are en route. The patient dies, and the physician informs the wife of her husband's death. When asked to provide consent for an autopsy, his wife replies, "No. He hated being in the hospital, and I don't want him to be cut up." Which of the following is the most appropriate next step?

O A) Ask the parents to consent to the autopsy when they arrive
O B) Ask the wife again and explain the potential benefits of an autopsy
O C) Notify the medical examiner to mandate an autopsy
O D) Perform genetic testing on a postmortem blood sample

CCCCCCCCCCCCCCC



9. A 37-year-old man comes to the physician for a routine follow-up examination. Eight years ago, he had the gradual onset of symptoms of schizophrenia without any apparent precipitating factors. He lives alone and is able to care for himself. He has no history of depression. He has had two brief hospitalizations for medication adjustment. His only medication is risperidone. Physical examination shows no abnormalities. Mental status examination shows a mildly constricted affect. He is concerned about his prognosis. Which of the following historical factors is most predictive of a good prognosis for this patient?
A) Absence of depression
B) Absence of precipitating factors for symptoms
C) Gradual onset of symptoms
D) Late onset of symptoms
E) Living independently

DDDDDDDDDDDDDDDDDDD



10. A previously healthy 4-year-old girl is brought to the emergency department because of a 1-day history of fever, swelling of the right knee, and refusal to walk. Her temperature is 38.3°C (100.9°F), blood pressure is 90160 mmHg, pulse is 115/min, and respirations are 24/min. Examination of the right knee shows erythema, swelling, warmth, and tenderness, range of motion is decreased.
Laboratory studies show:
Hematocrit 34%
Leukocyte count 15,000/mm3
Segmented neutrophils 80%
Bands 10%
Lymphocytes 10%
Platelet count 400,000/mm3
Erythrocyte sedimentation rate 85 mm/h

An x-ray film of the right knee shows a joint effusion. Which of the following is the most appropriate next step in management?

O A) Rest and avoidance of activity
O B) Intra-articular corticosteroid therapy
O C) Nonsteroidal anti-inflammatory drug therapy
O D) Cast immobilization
0 E) Arthrocentesis

EEEEEEEEEEEEEEEEEEEEE

11. A 26-year-old woman, gravida 3, para 1, aborta 1, at 33 weeks' gestation is brought to the emergency department because of a 3-hour history of uterine pain and contractions and a 2-hour history of vaginal bleeding. Examination shows an extremely tender uterus. Dark blood is seen in the vagina. The fetal heart rate is 1301min. Which of the following is the most likely diagnosis?
O A) Abruptio placentae
O B) Chorioamnionitis
O C) Placenta previa
O D) Placental tumor
0 E) Uterine rupture


AAAAAAAAAAAAAAAAAA




12. A 57-year-old woman comes for a follow-up examination. She has a 15-year history of type 2 diabetes mellitus and hypertension well controlled with an angiotensin-converting enzyme (ACE) inhibitor. Examination shows no flank tenderness. The kidneys are not palpable. Laboratory studies show.
Serum creatinine 1.9 mg/dL
Urine
Protein 1+
WBC 5-7/hpf
Which of the following is the most appropriate next step in management to minimize progression of her renal disease?
o A) High-phosphate diet
p B) Low-protein diet
O C) Daily corticosteroid therapy
p D) Ibuprofen therapy
p E) Sodium chloride supplementation


BBBBBBBBBBBBBBBBB



13. A 22-year-old man is brought to the emergency department 20 minutes after he sustained a gunshot wound to the abdomen. On arrival, administration of oxygen by face mask and fluid resuscitation with 2 L of lactated Ringer's solution are begun. He is alert but unable to move his lower extremities. His temperature is 37.4°C (99.3°F), blood pressure is 140/80 mm Hg, pulse is 105/min, and respirations are 14/min. The pupils are equal and reactive to light. Examination shows no jugular venous distention. Breath sounds are equal bilaterally, and heart sounds are normal. There is a single gunshot wound to the left of the umbilicus that is not bleeding. No exit wound can be found. The abdomen is nondistended and soft, there is mild tenderness around the wound. Bowel sounds are normal. Rectal examination shows no sphincter tone. Sensation is absent below the L2 level. Test of the stool for occult blood is positive. Insertion of a nasogastric tube yields clear fluid, and insertion of a urinary catheter yields clear urine. A urine dipstick is negative for blood. X-ray films of the chest, abdomen (supine and decubitus), and pelvis show a small caliber bullet just to the right of the spine at L2. No other abnormalities are seen. Which of the following is the most appropriate next step in diagnosis?

O A) Ultrasonography of the abdomen
O B) CT scan of the abdomen
O C) Colonoscopy
O D) Diagnostic peritoneal lavage
O E) Arteriography
O F) Laparotomy

FFFFFFFFFFFFFFFFFFFF

14. A 67-year-old woman is brought to the emergency department because of the inability to move the right side of her body since awakening this morning. She has type 2 diabetes mellitus. Her blood pressure is 170/95 mm Hg. Examination shows dysarthric speech, but language function is normal. Cranial nerves are intact except for moderate weakness of the right lower face. She is unable to move the right upper or lower extremities. Sensation is intact. There is hyperreflexia of the right upper and lower extremities, and Babinski's sign is present on the right. Which of the following is the most likely location of the lesion?
O A) Basal ganglia
O B) Brain stem
O C) Cerebellum
O D) Cervical nerve root
O E) Frontoparietal cortex
O F) Internal capsule

FFFFFFFFFFFFFFFFFFFFFFF


15. A previously healthy 20-month-old girl is brought to the emergency department because of vomiting and diarrhea for 5 days, her stools have been streaked with blood and mucus. Her oral intake and urine output have decreased during this period. She appears lethargic. Her temperature is 38°C (100.4°F), blood pressure is 60140 mm Hg, pulse is 1501min, and respirations are 401min. Examination shows dry mucous membranes. The remainder of the examination shows no abnormalities. Capillary refill time is 4 sec. Laboratory studies show:
Hematocrit 39%
Leukocyte count 6500/mm3
Segmented neutrophils 35%
Lymphocytes 65%
Platelet count 200,000/mm3
Serum
Na` 131 mEg1L
Cl- 94 mEq/L
K` 3.6 mEq/L
HC03 12 mEq/L
Urea nitrogen (BUN) 45 mg/dL
Creatinine 1.5 mg/dL
The most appropriate next step in management is administration of which of the following?

O A) Subcutaneous epinephrine
O B) Intravenous corticosteroids
O C) Intravenous immune globulin
O D) Intravenous sodium bicarbonate
O E) Intravenous 0.9% saline
0 F) Transfusion of packed red blood cells

EEEEEEEEEEEEE

16. A 57-year-old man comes to the physician because of a 6-month history of bloating, upper abdominal discomfort, and bulky, foul-smelling stools. He has had a 9-kg (20-Ib) weight loss during this period. He has a 30-year history of heavy alcohol use and has had multiple hospitalizations for acute pancreatitis. He is 178 cm (5 ft 10 in) tall and weighs 54 kg (119 Ib), BMI is 17 kglm2. The remainder of the examination shows no abnormalities. His serum glucose level is 130 mgfdL, and fecal fat (on a diet of 100 grams of fat daily) is 11 gramslday (N=6); serum electrolyte levels are within the reference range. d-xylose absorption testing shows no abnormalities. An xray film of the abdomen shows diffuse calcifications in the upper abdomen. A deficiency of which of the following is the most likely
cause of these findings?
O A) Amylase
O B) Bile salts
O C) Cholecystokinin
O D) Insulin
O E) Lipase
0 F) Secretin


EEEEEEEEEEEEEEEEE


17. A 27-year-old primigravid woman at 39 weeks' gestation has been in labor for 10 hours. Over the past 3 hours, the cervix has remained 90% eff aced and 8 cm dilated. Contractions occur every 8 minutes. The fetal heart rate is 1401min. Her temperature is 37°C (98.6°F), blood pressure is 120180 mm Hg, pulse is 80/min, and respirations are 20/min. The vertex is left occiput anterior and at 0 station. Which of the following is the most appropriate next step in management?

O A) Oral administration of a barbiturate
O B) Augmentation with oxytocin
O C) Rotation of the fetal head to occiput posterior
O D) Vacuum delivery
O E) Cesarean delivery

BBBBBBBBBBBBBBBBBBBBBB

18. A 55-year-old man with hypertension comes to the emergency department 6 hours after the acute onset of epigastric pain, nausea, and vomiting. He takes spironolactone and a calcium-channel blocking agent. His temperature is 38.3°C (101°F), blood pressure is 140180 mm Hg, pulse is 125Imin, and respirations are 22/min. Abdominal examination shows epigastric tenderness and guarding, there are no masses. Bowel sounds are decreased. Serum studies show a calcium level of 7.0 mg/dL, amylase activity of 7600 UIL, and lipase activity of 5650 U/L (N=10-150). Which of the following is the most likely cause of his symptoms?

O A) Calcium-channel blocking agent therapy
O B) Gallstones
O C) Hyperparathyroidism
O D) Spironolactone diuretic therapy
O E) Unrecognized abdominal trauma


BBBBBBBBBBBBBBBBBB

19. A 20-month-old boy is brought to the emergency department 10 minutes after being found unresponsive in the garage, there were numerous liquid containers spilled at his side. His skin is wet, he is drooling and has a clear nasal discharge. His temperature is 37.3°C (99.1°F), blood pressure is 95/65 mm Hg, pulse is 62/min, and respirations are 32/min. He is responsive only to deep pain. The pupils are pinpoint. Which of the following is most likely to confirm the diagnosis?

O A) Administration of glucose
O B) Administration of naloxone
O C) Measurement of erythrocyte cholinesterase activity
O D) Measurement of plasma lactate and pyruvate levels
O E) Measurement of serum electrolyte levels and osmolality


CCCCCCCCCCCC

20. A 9-year-old boy is brought to the physician because of a 4-day history of pain in a tooth. He has had shaking chills during the past 2 days. Although he points to a tooth as the source of the pain, his dental examination 2 days ago showed no abnormalities. Ibuprofen relieves his pain for a short time. He has no history of serious illness. Growth and development are appropriate for age. He is in severe distress. His temperature is 39°C (102.2°F), pulse is 1201min. and blood pressure is 90160 mm Hg. The right side of the face, including the area around the right periorbital tissue, is edematous and mildly erythematous. On percussion of the upper jaw inside the oral cavity, there is severe tenderness of the right upper jaw and the area under the right eye. Examination of the oral cavity shows no other abnormalities. Both tympanic membranes are clear. Which of the following is the most likely source of infection?


O A) Pharynx
O B) Right maxillary sinus
O Q Right middle ear
O D) Right periorbital region
O E) Right upper molar

BBBBBBBBBBBBBBBB



21. Three hours after biopsy of a 2-cm mass in the upper outer quadrant of the left breast, a 45-year-old woman has pain and swelling of the left breast. She has no history of bleeding problems and takes no medications. Examination shows an expanding hematoma. A biopsy specimen shows a fibroadenoma. Which of the following is the most likely diagnosis?

O A) Factor VIII deficiency
O B) Factor IX deficiency
O C) Inadequate hemostasis
O D) Platelet dysfunction
O E) Thrombocytopenia


AAAAAAAAAAAAAAAAA

22. A previously healthy 23-year-old woman comes to the physician because of pain and burning with urination for 3 days and loss of urine several times over the past 24 hours. She has not had fever or chills. Examination shows no flank or costovertebral angle tenderness. Urinalysis shows leukocytes too numerous to count and numerous rod-shaped bacteria. Which of the following is the most likely mechanism of this patient's urinary symptoms?

A) Decreased bladder muscle tone
B) Delayed neuromuscular maturation
C) Ectopic insertion of the ureters
D) Trigonal inflammation
E) Uninhibited detrusor muscle conditions


DDDDDDDDD

23. A 57-year-old man is admitted to the hospital because of respiratory failure due to an exacerbation of chronic obstructive pulmonary disease. Thirty-six hours after intubation and initiation of mechanical ventilation, his oxygen saturation suddenly decreases from 96%
to 78% without changing any ventilator settings. His FIO 2 is 40%, ventilatory rate is 12/min, and tidal volume is 800 mL. There is no positive end-expiratory pressure. He is conscious and in mild respiratory distress. His temperature is 39.2°C (102.6°F), blood pressure is 110/70 mm Hg, pulse is 1201min, and respirations are 381min. Examination shows no inflammation at the skin site where the central catheter was inserted. Breath sounds on the left are present, but decreased in volume. Breath sounds on the right are absent, and there is hyperresonance over the right hemithorax. There is no dullness to percussion on either side. Cardiac examination shows distant heart sounds but no rubs, murmurs, or gallops. An ECG shows sinus tachycardia and diffuse low voltage. Which of the following is the most likely explanation for these findings?

p A) Hemothorax
O B) Improper placement of the endotracheal tube
O C) Mucous plug
O D) Pericardial tamponade
p E) Pneumothorax
0 F) Sepsis

EEEEEEEEEEEEE

24. A 70-year-old woman is hospitalized for evaluation of a single, transient episode of right-sided weakness. She has type 2 diabetes mellitus controlled with diet. Seven years ago, she underwent lumpectomy and radiation therapy for localized breast cancer and has had no evidence of recurrence. Examination shows a left carotid bruit. Arteriography shows 80%-90% stenosis of the left internal carotid artery in a segment just distal to where it branches from the common carotid artery. On admission, her serum glucose level is 310 mgfdL, after adequate hydration, her serum glucose level decreases to 180 mgldL. Serum electrolyte levels are within normal limits. Which of the following is the most appropriate recommendation to prevent cerebral infarction in this patient?
O A) Daily aspirin therapy only
O B) Insulin therapy only
O C) Low-dose heparin therapy only
O D) Warfarin therapy only
O E) Endarterectomy and daily aspirin therapy
O F) Endarterectomy and low-dose heparin therapy
O G) Endarterectomy and warfarin therapy

EEEEEEEEEEEEEEEEEEEEEEEEE



25. A previously healthy 10-year-old boy is brought to the emergency department because of increasing lethargy over the past 8 hours. He has had abdominal pain for 16 hours and has vomited nine times during this period. He has not had diarrhea or fever. He is sleepy but arousable to pain. His temperature is 38.5°C (101.3°F), blood pressure is 92/60 mm Hg, pulse is 112/min, and respirations are 24/min. The mucous membranes are dry. The lungs are clear to auscultation. No murmurs are heard. There is diffuse tenderness to palpation of the abdomen with guarding but no rebound, masses, or hepatosplenomegaly. Bowel sounds are active in all quadrants.

Capillary refill time is 3-4 sec. Laboratory studies show.
Hematocrit 39%
Leukocyte count 20,0001mm3
Segmented neutrophils 70%
Bands 8%
Lymphocytes 7%
Monocytes 15%
Platelet count 300,000/mm3

Serum
Na' 142 mEgIL
CI- 106 mEq/L
K' 3.3 mEgIL
HCO3 10 mEq/L
Urea nitrogen (BUN) 26 mg/dL
Creatinine 0.5 mg/dL
Glucose 350 mgfdL
Amylase 60 U1L
Urine
pH 4.5
Specific gravity 1.029
Glucose large
Ketones large
WBC 0-3/mm3
RBC 0-3/mm3


Which of the following is the most likely diagnosis?
A) Appendicitis
B) Diabetic ketoacidosis
C) Gastroenteritis
D) Pancreatitis
E) Pyelonephrltis

BBBBBBBBBBBBBBBBBBBBBBBBBBB

26. A 37-year-old man comes to the physician 2 days after the sudden onset of severe low back pain. The pain began when he was lifting heavy boxes. He is otherwise healthy and takes no medications. Examination shows paraspinal muscle tenderness and spasm. After 5 days of bed rest and oxycodone therapy, his back pain resolves. Which of the following measures is most likely to prevent a recurrence of his symptoms?

A) Exercise program
B) Use of a back brace
C) Spinal manipulation
D) Nonsteroidal anti-inflammatory drug therapy
E) Use of muscle relaxants

AAAAAAAAAAAAAAAAAAAAAAA

27. Five days after undergoing an emergency appendectomy for a perforated appendicitis, a 47-year-old woman has abdominal pain, nausea, and vomiting. She is pale and diaphoretic. Her temperature is 39°C (102.2°F), blood pressure is 82144 mmHg, pulse is 115Imin, and respirations are 181min. There is nojugularvenous distention. The lungs are clear to auscultation. Abdominal
examination shows mild distention and tenderness in the right lower quadrant. The upper and lower extremities are warm. Rectal examination shows a tender mass in the right pelvis extrinsic to the rectum. Which of the following is the most likely set of hemodynamic findings in this patient?
Preload Cardiac Contractility Afterload
O A) D D D
O B) D I D
O C) D I I
O D) I D I
O E) I I D
O F) I I I


BBBBBBBBBBBBBBB



28. A 67-year-old woman is brought to the physician in the summer by her son because of fatigue, dizziness, and confusion for 1 day. She lives alone and does her own shopping and housekeeping, but she has stayed inside for the past 3 days because the hot weather has made her unsteady on her feet. She has hypertension treated with hydrochlorothiazide. Her temperature is 39.2°C (102.6°F), pulse is 116/min, respirations are 18/min, and blood pressure is 90/65 mm Hg. Examination shows a supple neck. The lungs are clear to auscultation. Cardiac examination shows no abnormalities. Cranial nerves and muscle strength are intact, and deep tendon reflexes are normal. There is no tremor when she slides the heel of one foot down the opposite shin. She is oriented to person but not to place or time and cannot remember her telephone number. Serum studies show:

Na' 157 mEq/L
Cl- 117 mEq/L
K' 3.7 mEq/L
HCO3 30 mEgIL
Urea nitrogen (BUN) 36 mg/dL
Creatinine 1.2 mg/dL
Which of the following is the most appropriate next step in management?
O A) MRI of the brain
O B) Administration of broad-spectrum antibiotics
O C) Administration of dopamine
O D) Administration of 0.9% saline
0 E) Lumbar puncture


DDDDDDDDDDDDDDDDDD


29. A 16-month-old boy is brought to the emergency department because of 2- to 3-minute episodes of acute abdominal pain over the past 6 hours. During the episodes, he turns pale, screams, and draws up his knees. He has also passed one bloody stool. A mass is palpated in the right upper quadrant of the abdomen. Which of the following is the most appropriate next step in management?
O A) Nothing by mouth, administration of fluids and antibiotics, intravenously
O B) Water-soluble contrast enema
O C) CT scan of the abdomen
O D) Colonoscopy
O E) Laparotomy


BBBBBBBBBBBBBBBBBBBBBBBB

30. A 15-year-old girl comes to the physician because she is concerned about her risk for gonorrhea. Her sexual partner was recently diagnosed with this disease, and they had intercourse before he started treatment. After testing for Neisseria gonorrhoeae and Chlamydia trachomatis, which of the following is the most appropriate next step in management?

A) Treatment only if symptomatic
B) Treatment only if cultures are positive
C) Treatment for N. gonorrhoeae only, based on history of exposure
D) Treatment for N. gonorrhoeae and C. trachomatis, based on history of exposure

DDDDDDDDDDDDDDDDDD


31. A 47-year-old man is brought to the physician by his sister because she is concerned about his 10-year infatuation with a famous television actress. He is convinced that the actress is in love with him even though he has never met or spoken with the actress. He has no history of major depressive disorder or manic symptoms. Physical examination shows no abnormalities. Mental status examination is normal except for his infatuation. There is no evidence of hallucinations. Which of the following is the most likely diagnosis?

O A) Delusional disorder
O B) Exhibitionism
O C) Narcissistic personality disorder
O D) Schizoaff ective disorder
0 E) Schizophrenia


AAAAAAAAAAAAAAAAAAA


32. A 57-year-old woman comes in November for an annual health maintenance examination. She has chronic bronchitis treated twice daily with a G32 adrenergic agonist. Since her last visit 1 year ago, she has not had any changes in daily cough, sputum production, or exercise tolerance. She has had a 2.3-kg (5-Ib) weight loss due to decreased appetite since her last examination. She currently
weighs 54 kg (120 lb) and is 157 cm (62 in) tall, BMI is 22 kgfm2. Her blood pressure is 130/80 mm Hg, pulse is 961min, and respirations are 201min. Examination shows a barrel-shaped chest. Breath sounds are decreased throughout all lung fields, scant rhonchi are heard at both lung bases. Pulse oximetry on room air shows an oxygen saturation of 92%. An x-ray film of the chest shows
hyperinflation of the lungs, unchanged from 1 year ago. Pulmonary function tests show severe obstruction and are also unchanged from her last examination. Which of the following is the most appropriate recommendation for this patient?

p A) Measurement of daily caloric intake
p B) PPD skin testing
o C) Sputum culture
O ID) Pulmonary rehabilitation program
0 E) Influenza virus vaccine

EEEEEEEEEEEEEEEEEEEEEE

33. A 55-year-old woman has had two acute episodes of vertigo associated with nausea and decreased hearing and tinnitus in the left ear over the past 24 hours. The episodes lasted approximately 3 hours. The second episode occurred this morning, and she still has
ringing and decreased hearing in the left ear. She is otherwise healthy and takes no medications. Physical examination and an MRI of the brain show no abnormalities. Audiometry shows low-frequency sensorineural hearing loss. Which of the following is the most likely
diagnosis?
O A) Acoustic neuroma
O B) Meniere's disease
O C) Multiple sclerosis
O D) Syphilis
O E) Viral labyrinthitis (sudden onset, preceded with infection, vertigo lasting for days. See Kaplan IM p 307)

BBBBBBBBBBBBBBBBBBBBBB



34. A 10-year-old girl has had fatigue and pallor for 3 weeks. Six weeks ago, she underwent aortic valve replacement with a mechanical prosthesis. She is receiving digoxin therapy. Her temperature is 37°C (98.6°F), blood pressure is 100160 mmHg, pulse is 1151min, and respirations are 241min. A grade 216, systolic ejection murmur is heard along the left sternal border, and a grade 2/6, decrescendo diastolic murmur is heard along the left sternal border to the cardiac apex. The liver and spleen are palpable. Her hemoglobin level is 4.6 gldL. A blood smear shows schistocytes. Which of the following is the most likely cause of the anemia?

p A) Autoimmune hemolytic anemia
o B) Digoxin-induced hemolytic anemia
p C) Glucose 6-phosphate dehydrogenase deficiency
O D) Hemolytic-uremic syndrome
0 E) Periprosthetic valve leak

EEEEEEEEEEEEEEEEEE



35. A 62-year-old man is brought to the emergency department because of a 2-hour history of left anterior chest pain that began after he finished raking leaves. Initially, he thought that he had strained a chest muscle. Rest did not relieve the pain, and it began to radiate to his back. He has a 20-year history of hypertension treated with (3-adrenergic blocking agents, he is not always compliant with his medication regimen. His temperature is 37°C (98.6°F), blood pressure is 1751100 mm Hg, pulse is 951min and regular, and respirations are 18/min. The lungs are clear to auscultation. A grade 216 diastolic murmur is heard at the cardiac base when the patient leans forward. Radial pulses are stronger on the left than on the right. An ECG shows sinus tachycardia. Which of the following is the most likely diagnosis?
O A) Costochondritis
O B) Dissecting aortic aneurysm
O C) Gastroesophageal reflux disease
O D) Myocardial infarction
O E) Pericarditis
O F) Pulmonary embolism
O G) Stable angina pectoris
O H) Unstable angina pectoris

BBBBBBBBBBBBBBBBBBBBB

36. A 30-year-old man is brought to the emergency department 30 minutes after the onset of increasingly severe, tearing anterior chest pain that radiates to the back between the scapulae. His temperature is 37.1°C (98.8°E), blood pressure is 100140 mm Hg, pulse is 130Imin, and respirations are 30/min. He appears pale. His upper and lower extremities are cold, clammy, long, and thin. The arches of the feet are high. The left radial pulse is absent, other peripheral pulses are palpable. There is a grade 2/6, blowing diastolic murmur along the right sternal border. An x-ray film of the chest shows a markedly widened superior mediastinum. An ECG shows nonspecific
ST-T-wave changes in the lateral precordial leads. Which of the following underlying mechanisms best explains this patient's disease?
O A) Atherosclerosis
O B) Cystic medial necrosis
O C) Mycotic infection
O D) Rheumatic aortitis
0 E) Syphilitic infection

BBBBBBBBBBBBBBBBBBBBB

The response options for the next two items are the same. You will be required to select one answer for each item in the set. For each patient with a skin lesion, select the most likely diagnosis.
O A) Basal cell carcinoma
O B) Blue nevus
O C) Epidermoid cyst
O D) Keratoacanthoma
O E) Melanoma
O F) Seborrheic keratosis
O G) Squamous cell carcinoma



Picture here

AAAAAAAAAAAAAAAAA




37. A 42-year-old man has had a 2-cm mass on the temporal area above the left eye for 4 years. A photograph is shown. It recently has
become erythematous and tender. The lesion is smooth and easily movable with no pigmentation or ulceration.



For each patient with a skin lesion, select the most likely diagnosis.

A) Basal cell carcinoma
B) Blue nevus
C) Epidermoid cyst
D) Keratoacanthoma
E) Melanoma
F) Seborrheic keratosis
G) Squamous cell carcinoma




FFFFFFFFFFFF



38. An 82-year-old man has had multiple, pigmented, verrucous lesions on the back for 10 years. A photograph is shown. The lesions appear wax-like.




39. A 27-year-old woman, gravida 2, para 1, at 40 weeks' gestation comes to the physician 24 hours after the acute onset of severe vulvar burning on urination and a clear yellow vaginal discharge. She has no history of similar symptoms. Examination shows enlarged, tender nodes in the groin. There are several small, 2- to 3-mm ulcerations and several vesicular lesions in the vulvar area. Cultures from these areas and the cervix and a urine culture are obtained. This patient is at increased risk for which of the following pregnancy outcomes?
O A) Chorioamnionitis
O B) Fetal growth restriction
O C) Fetal malformations
O D) Intrapartum transmission of herpes
O E) Premature rupture of the membranes

DDDDDDDDDDDDDDDDDDD

40. A previously healthy 16-year-old girl comes to the physician because of a headache, nausea, and one episode of vomiting over the past 2 days. She has had a constant headache for 2 hours; she describes it as throbbing and located over the left eye. The symptoms are only partially relieved with ibuprofen therapy. There is no visual impairment. She has had intermittent headaches over the past year. Her menstrual period began 2 days ago. She is sexually active with one partner and uses condoms for contraception. Her temperature is 36.7°C (98°F), blood pressure is 114/70 mm Hg, pulse is 104/min, and respirations are 18/min. Neurologic examination shows no abnormalities. Which of the following is the most appropriate next step in management?

O A) Testing for Neisseria gonorrhoeae
O B) CT scan of the head
O C) Administration of acetazolamide
O D) Administration of sumatriptan
O E) Lumbar puncture



DDDDDDDDDDDDDDDDDDDD



41. A 42-year-old woman comes to the physician because of a 4-day history of pain and swelling of her left wrist. There is no history of trauma. Treatment with ibuprofen has improved her symptoms. She has type 1 diabetes mellitus well controlled with insulin. Her temperature is 38°C (100.4°F). Examination of the left wrist shows swelling on the dorsal and volar aspects, slight warmth, erythema, and tenderness to palpation. Range of motion of the wrist is to 45 degrees of flexion and extension and elicits pain. An x-ray of the left wrist shows chondrocalcinosis. Which of the following is the most likely cause of these findings?
O A) Amyloid infiltration of the flexor retinaculum
O B) Avascular necrosis of the distal radius
O C) Calcium pyrophosphate arthritis
O D) Gonococcal tenosynovitis
O E) Monosodium urate synovitis


Cccccccccccccccccccccc



42. A 27-year-old woman at 28 weeks' gestation comes for her first prenatal visit. She was diagnosed with HIV infection 1 year ago but elected not to receive any treatment. Examination shows an oral herpes simplex lesion. The uterus is consistent in size with a 28-week gestation. Her hematocrit is 37%, and CD4+ lymphocyte count is 275/mm3 (Normal_500). Which of the following measures is most likely to prevent HIV transmission to the newborn?

A) Zidovudine (AZT) and acyclovir therapy for the mother until delivery and no treatment for the newborn
B) AZT therapy for the mother throughout labor and no treatment for the newborn
C) AZT therapy for the mother until delivery, and AZT therapy for the newborn for 6 weeks
D) No treatment for the mother and AZT therapy for the newborn for 1 week
E) Immediate cesarean delivery followed by AZT therapy for 6 weeks for the newborn

ccccccccccccccccc




43. A 12-year-old girl has had episodes of facial grimacing and the inability to sustain a grasp over the past 10 days. She has become increasingly clumsy and has had episodes of tearfulness over the past month. Teachers have noted deterioration in her handwriting. Two months ago, she had a fever and sore throat for 2 days that resolved spontaneously. Examination shows facial grimacing and
spontaneous nonpurposeful movements of the face, trunk, and upper extremities. Her leukocyte count and erythrocyte sedimentation rate are within normal limits. Which of the following is the most likely diagnosis?

O A) Ankylosing spondylitis
O B) Dermatomyositis
O C) Erythema nodosum
O D) Henoch-Schonlein purpura
O E) Juvenile rheumatoid arthritis
O F) Mucocutaneous lymph node syndrome (Kawasaki disease)
O G) Polyarteritis nodosa
O H) Rheumatic fever
O I) Systemic lupus erythematosus
O J) Systemic sclerosis (scleroderma)
0 K) Wegener's granulomatosis


HHHHHHHHHHHHHHHHHHHHHHHH




44. A 62-year-old man with a 20-year history of alcoholism is admitted to the hospital for treatment of alcoholic hepatitis. He appears disheveled and malnourished. He is 198 cm (6 ft 6 in) tall and weighs 70 kg (155 lb). BMI is 18 kglm2. Examination shows jaundice and temporal wasting. Scattered rhonchi are heard throughout all lung fields. Cardiac examination shows no abnormalities. Bowel sounds are normal. The liver span is 16 cm. Sensation to pinprick and light touch is decreased over the feet. Deep tendon reflexes are decreased at the ankles. Laboratory studies show

Hematocrit 33%
Platelet count 145,0001mm3
Serum
Na+ 131 mEq/L
Cl- 92 mEg1L
K+ 3.1 mEq/L
HC03 26 mEgIL
Mg2+ 0.8 mEgIL
Ca 2+ 5.8 mg1dL
Urea nitrogen (BUN) 6 mgfdL
Creatinine 0.8 mgldL

Test of the stool for occult blood is positive. Which of the following is the most likely mechanism of this patient's hypocalcemia?
O A) Chronic metabolic acidosis
O B) Hypomagnesemia
O C) Impaired hydroxylation of vitamin D
O D) Primary hyperparathyroidism
O E) Renal resistance to parathyroid hormone

Bbbbbbbbbbbbbbbbbbbbbb




45. An 82-year-old woman with a 20-year history of urinary incontinence has had a mild exacerbation of her symptoms over the past 3 months. Urine loss generally occurs when she is carrying out daily activities such as shopping or driving and is not affected by coughing or sneezing. She underwent appendectomy at the age of 24 years. She has one daughter. She takes no medications. Pelvic examination shows an atrophic cervix without a palpable uterus or an adnexal mass. Laboratory studies show:
Hemoglobin 13 g/dL
Serum
Na* 140 mEq/L
Cl- 105 mEq/L
K+ 4.5 mEq/L
HC03 25 mEq/L
Urea nitrogen (BUN) 15 mg/dL
Glucose 120 mg/dL
Creatinine 1.1 mg/dL
Urine
Epithelial cells 5-10
Glucose negative
WBC 0-1/hpf
Bacteria occasional
Which of the following is the most likely cause of this patient's urinary incontinence?
O A) Detrusor instability
O B) Hyperglycemia
O C) Neurogenic bladder
O D) Obstructive uropathy
O E) Urinary tract infection

AAAAAAAAAAAAAAAAAAAAAAAA



46. A 47-year-old woman comes to the physician because of swelling of her left arm for 3 weeks. She underwent lumpectomy, axillary node dissection, and radiation therapy for T1 NO cancer of the left breast 11 months ago. Five months ago, mammography showed no new abnormalities. She plays tennis three times weekly. Her temperature is 37°C (98.6°F), pulse is 72/min, respirations are 12/min, and blood pressure is 130/82 mm Hg. Examination of the left upper extremity shows edema to twice the normal size. Which of the following is the most appropriate next step in management?

O A) Heparin therapy
O B) Tamoxifen therapy
O C) Adjuvant chemotherapy
OD) Lymphatic compression therapy
0 E) Thrombectomy


ddddddddddddddddddddddddd





National Board of Medical Examiners
Comprehensive Clinical Science Self-Assessment


Exam Section 4: Item 1 of 46

1.A 27-year-old woman is hospitalized after sustaining bilateral femoral shaft fractures in a motor vehicle collision. Twenty minutes after transfusion of 1unit of packed red blood cells, she develops back and flank pain and hematuria. Her temperature is 38.5°C (101.3°F). Which of the following is the most likely cause of these symptoms?

O A) ABO incompatibility
O B) Contamination of the transfused blood
O C) Fat embolism syndrome
O D) Reaction to leukocyte antigens in the transfused blood
O E) Retroperitoneal bleeding

AAAAAAAAAAAAAAAA




2. An asymptomatic 57-year-old man comes to the physician before beginning a jogging program. He has a sedentary lifestyle and has smoked two packs of cigarettes daily for 30 years. His blood pressure is 150188 mm Hg. Examination shows no other abnormalities. An ECG shows normal findings. His serum cholesterol level is 250 mgfdL. Which of the following is the most appropriate next step in management?

O A) Referral to a supervised exercise program
O B) Ambulatory blood pressure monitoring
O C) Ambulatory ECG monitoring
O D) Exercise stress test
0 E) Echocardiography


AAAAAAAAAAAAA


3. A 16-year-old boy with cystic fibrosis is brought to the physician because of shortness of breath and ankle edema over the past 2 days. He is below the 5th percentile for height and weight. His temperature is 37°C (98.6°F), blood pressure is 130/80 mm Hg, pulse is 82/min, and respirations are 251min and labored. Examination shows mild perioral cyanosis, jugular venous distention, and bilateral pitting ankle edema. There is mild right upper quadrant tenderness. S1 is normal, and S2 is increased. Which of the following is the most likely cause of this patient's symptoms?

O A) Decreased systemic arterial pressure
O B) Increased plasma colloid oncotic pressure
O C) Increased pulmonary vascular resistance
O D) Portal hypertension
O E) Reduced left ventricular ejection fraction

ccccccccccccccccccccccc


4. A 27-year-old woman, gravida 1, pars 1, comes to the physician because she is concerned about her risk for ovarian cancer. Her mother was diagnosed with ovarian cancer during her 60s. There are no other family members on either the maternal or paternal side with cancer. The patient uses a diaphragm for contraception, and she and her husband are unsure if they want more children. She drinks one to two glasses of wine each day. She is 163 cm (5 ft 4 in) tall and weighs 63 kg (140 Ib), BMI is 24 kg/mz. The remainder of the examination shows no abnormalities. Which of the following is the most appropriate step to decrease this patient's risk for ovarian cancer?

A) Abstinence from alcohol
B) Low-fat diet
C) Weight loss
D) Measurement of serum CA 125 level every 6 months
E) Use of an oral contraceptive
F) Removal of ovaries

E
EEEEEEEEEEEEEEEEE

5. A 1-year-old girl is brought to the physician by her grandmother because of a 1-week history of crying immediately before and during bowel movements. She is otherwise asymptomatic. Examination shows no abnormalities except for an anal fissure. Which of the following is the most appropriate initial step in management?

O A) Notification of child protective services
O B) Stool softener therapy
O C) Anal dilatation in the office
O D) Anal dilatation with general anesthesia
O E) Excision of the fissure
0 F) Sphincterotomy

BBBBBBBBBBBBBBBBBB


6. A 47-year-old man comes to the physician because of swelling of his feet for 1 month. He is otherwise asymptomatic. His last visit to a physician was 30 years ago. He has smoked two packs of cigarettes daily for 30 years and drinks three to four beers each night. He weighs 82 kg (180 lb) and is 185 cm (73 in) tall. His temperature is 37.2°C (99°F), blood pressure is 152192 mm Hg, pulse is 80/min,
and respirations are 121min. Examination shows no other abnormalities except for 2+ pedal edema and decreased sensation to light touch over the distal lower extremities. Serum studies show a glucose level of 240 mg1dL, creatinine level of 2.7 mgldL, and total cholesterol level of 252 mg/dL. Urine dipstick shows 3+ protein. Which of the following is most likely to have prevented this condition?

O A) Abstinence from alcohol
O B) Lowering of serum cholesterol level
O C) Low-protein diet
OD) Tight glycemic control
0 E) Weight loss

DDDDDDDDDDDDDDDDDDDDDDDDDDD


7. A 2-month-old infant is brought to the emergency department by paramedics after his mother found him not breathing in his crib. He is apneic and cyanotic; no pulses are palpable, but he is warm. After manually stabilizing his neck and positioning the airway, which of the following is the most appropriate next step in management?
O A) Positive pressure ventilation and oxygenation
O B) Two chest thrusts followed by chest compressions
O C) Establishing vascular access
O D) Administration of epinephrine
0 E) Electrical defibrillation

I would go for A.




8. A 43-year-old woman comes to the physician because of increasingly severe headaches for 6 weeks. She has a 15-year history of chronic headaches and has taken at least 3 g of acetaminophen daily for 5 years. She also uses aspirin and ibuprofen frequently and has used other analgesics over the past 15 years. Her blood pressure is 166198 mm Hg. Examination shows no other abnormalities.
This patient is at greatest risk for which of the following?

O A) Acute tubular necrosis
O B) Chronic interstitial nephritis
O C) Hepatorenal syndrome
O D) Membranoproliferative glomerulonephritis
O E) Rapidly progressive glomerulonephritis

BBBBBBBBBBBBBBBBBBBBBB

9. A 25-year-old woman comes to the physician because of low-grade fevers, headache, malaise, and diffuse arthralgias for 6 weeks. Use of over-the-counter nonsteroidal anti-inflammatory drugs has not provided consistent relief of the headache and arthralgias. Bilateral carotid bruits are heard that are greater on the left than on the right. There are decreased bilateral brachial and radial pulses. Erythrocyte sedimentation rate is 110 mm/h. Which of the following is the most likely diagnosis?

O A) Polymyalgia rheumatica (main s/s is stiffness, rarely in young woman
Reply
#6
First post is NBME 2 n 2nd post is last 2 blocks of NBME 3 i guess....
Reply
#7
Thanks friend and good luck!
Reply
#8
thanks hz28 if u get 4 sec of form 6 pl also post it good luck
Reply
« Next Oldest | Next Newest »


Forum Jump: